Skip to main content

Full text of "كتب في علم الميكروبيولوجي"

See other formats


BLACKWELL'S 



NDERGROUND CLINICAL VIGNETTES 



MICROBIOLOGY 
VOL II, 3E 



VIKAS BHUSHAN, MD 

University of California. San Francisco. Class of 1991 
Series Editor. Diagnostic Radiologist 

VISHAL PALL, MBBS 

Government Medical College. Chandigarh. India. Class of L990 
Series Editor, U. of Texas, Galveston, Resident in Internal Medicine & 
Preventive Medicine 

TAO LE, MD 

l nivciMH of California. San Francisco. Class of 1996 

HOANG NGUYEN, MD, MBA 
Northwestern University, Class of 2001 

SONAL SHAH, MD 

Ross University. Class of 2000 



b 



Blackwell 
Science 



CONTRIBUTORS 

Sandra Mnn 

University of Texas Medical Branch, Class of 2002 

Shalin Patel, MD 

McGraw Medical Center, Northwestern University, Resident in Internal 

Medicine 

Ashraf Zaman, MBBS 
New Delhi, India 

Vipal Soni, MD 

UCLA School of Medicine, Class of 1999 

FACULTY REVIEWER 

Warren Levinson, MD, PHD 

Professor of Microbiology and Immunology, UCSF School of Medicine 






© 2002 by Blackwell Science. Inc. 

Editorial Offices: 

Commerce Place. :5f>0 Main Street. Maiden. 

Massachusetts 02148, USA 

Osney Mead, Oxford OX2 DEL. England 
25 John Street, London WC1N 2BS. England 
23 Ainslie Place, Edinburgh EH3 6AJ. Scotland 

54 Universit) Street, Carlton, Victoria 3053, 
Australia 

Other Editorial Offices: 

Blackwell Wisseiischafts-Verlag GmbH, 

Kurfurstendamm 57, 10707 Berlin, Germany 
Blackwell Science KK. MG Kodenmacho Building. 

7-10 Kodenmacho Nihombashi. Chuo-ku, 

Tokyo 104, Japan 
Iowa State University Press, A Blackwell Science 

Company, 2121 S. State Avenue, Ames, Iowa 

500 1 -4-8300, USA 

Distributors: 

The Americas 

lilac kwell Publishing 

c/o AIDC 

P.O. Box 20 

50 Winter Spot I Lane 

Willision, VT 05495-0020 

(Telephone orders: 800-216-2522: 

fax orders: 802-864-7626) 
Australia 

Blackwell Science Pry, Ltd. 
54 University Street 

Carlton, Victoria 3053 
(Telephone orders: 03-9347-0300; 
fax orders: 03-9349-3016) 

(hi/side The Americas and Australia 
Blackwell Science, Lid. 
c/o Marslon Book Services. Ltd. 
P.O. Box 269 
Abingdon 
Oxon OX14 4YN 
England 

(Telephone orders: 44-01235-465500; 
fax orders: -14-01235-465555) 



Acquisitions: Laura DeYoung 

Development: Amy Nuttbrock 

Production: Lonia Hind and Shawn Gitsberger 

Manufacturing: Lisa Flanagan 

Mat kiting Manager: Kathleen Mulcahy 

Cover design bv Leslie Haimes 

Interior design by Shawn Girsberger 

rypesel In Tei hBooks 

Printed and bound by Capital City Press 

Blackwell's Underground Clinical Vignettes: 
Microbiology II. 3e 

ISBN 0-632-04549-3 

Printed in the United Stales of America 
02 03 04 05 5 4 3 2 1 

I lie Blackwell Science logo is a trade mark of 
Blackwell Science Ltd., registered at the United 
Kingdom Trade Marks Registry 

Library of Congress Cataloging-in-Publication Data 

Bhushan. Vikas. 

Blackwell's underground clinical vignettes. 

Microbiology / Author. Vikas Bhushan.- 3rd ed. 

p. : cm, - (Underground clinical vignettes) 
Rev. ed. of: Mirrobiology/Vikas Bhushan ... [el al.]. 
2nd ed, cl999-. ISBN 0-632-04547-7 (alk. paper) 

1. Medical microbiology- Case studies. 

2. Physicians - Licenses - United States - 
Examinations - Study guides. 

[DNLM: I. Microbiology - Case Report. 
2. Microbiology- Problems and Exercises. QW 18.2 
B575b 2002] I. Tide: Microbiology. II. Tide: 
Underground clinical vignettes. Microbiology. 
III. Microbiology. IV. Title. V. Series. 

QR46 .B465 2002 

616'.01'076-dc21 

2001004932 



.Ml rights reserved. No part of this book may be 
reproduced in any form or by any electronic or 
mechanical means, including inlormaiion storage 
and retrieval systems, without permission in writing 
from the publisher, except by a reviewei who may 
quote brief passages in a review. 



Notice 

IIk authors "I this volume have taken can' that the inhumation contained herein is accurate and compati- 
ble with the standards generally accepted at the time ol publication. Nevertheless, it is difficult to ensure 
thai all the information given is entirely accurate lor all circumstances. The publisher and authors do nol 
guarantee the contents ol this book .m<\ disclaim any liability, loss, or damage incurred as a consequence, 
direi ih or indirectly, ol the use and application ol any of die contents ol this vol nine. 






CONTENTS 



Acknowledgments x 

Preface to the 3rd Edition xiii 

How to use This Book xvi 

Abbreviations xvii 



Immunology 



Infectious Disease 



Anaphylaxis 

Chronic Granulomatous Disease 

Hereditary Angioedema 

Selective IgA Deficiency 

Severe Combined Immunodeficiency (SCID) 

X-Linked Hypogammaglobulinemia 

Histoplasmosis 

Human T-Cell Leukemia 

Virus Type 1 (HTLV-1) 

Inclusion Conjunctivitis 

Infectious Mononucleosis 

Influenza 

Jarisch-Herxheimer Reaction 

Legionella Pneumonia 

Leishmaniasis 

Leprosy — Lepromatous 

Leprosy— Tuberculoid 

Leptospirosis (Weil's Disease) 

Listeriosis 

Lyme Disease 

Lymphatic Filariasis 

Lymphogranuloma Venereum 

Malaria 

Measles 

Meningococcemia 

Mucormycosis 

Mumps 

Mycoplasma Pneumonia 

Necrotizing Fasciitis 

Nocardiosis 

Nosocomial Enterococcal Infection 

Onchocerciasis 

Overwhelming Postsplenectomy Infections 

Pasteurella multocida 

Plague 

Pneumococcal Pneumonia 

Pneumocystis carina Pneumonia 

Proctocolitis 



ESI 

m 



Psittacosis 

Pyogenic Liver Abscess 

Q Fever 

Rabies 

Rat Bite Fever 

Relapsing Fever 

Rocky Mountain Spotted Fever 

RSV Pneumonia 

Rubella — Congenital 

Rubella (German Measles) 

Salmonella Septicemia with Osteomyelitis 

Scabies 

Scarlet Fever 

Schistosomiasis 

Schistosomiasis — Urinary 

Shigellosis 

Shock — Septic 

Sporotrichosis 

Strongyloidiasis 

Subdiaphragmatic Abscess 

Syphilis — Congenital 

Syphilis — Primary EE1 

Syphilis — Secondary 

Syphilis — Tertiary (Tabes Dorsalis) 

Tetanus 

Tetanus Neonatorum 

Tick Paralysis 

Toxoplasmosis 133 

Trichinosis liftl 

Tuberculosis — Miliary |sf] 

Tuberculosis — Pulmonary 

Tularemia 

Typhoid Fever 

Urethritis — Nongonococcal 

Urinary Tract Infection (UTT) 

UTI with Staphylococcus saprophytics 

Varicella (Chickenpox) 

Visceral Larva Migrans 

Whooping Cough 

Yaws 

Yellow Fever 
Neonatology Listeria Meningitis in the Newborn KEI 

Nephrology/ Urology Acute Cystitis 

Epididymitis 

Orchitis 






Neurology 



Gynecology 



Obstetrics 
Orthopedics 



Poststreptococcal Glomerulonephritis 

Prostatitis — Acute 

Prostatitis — Chronic 

Pyelonephritis — Acute 

Brain Abscess 

Cysticercosis 

Herpes Simplex Encephalitis 

Japanese Encephalitis 

Lymphocytic Choriomeningitis (LCM) 

Meningitis — Bacterial (Adult) 

Meningitis — Bacterial (Pediatric) 

Meningitis — Cryptococcal 

Meningitis — Tubercular 

Poliomyelitis 

Progressive Multifocal 

Leukoencephalopathy 

Ramsay Hunt Syndrome 

St. Louis Encephalitis 

Subacute Sclerosing Panencephalitis 

Bacterial Vaginosis 

Breast Abscess 

Human Papillomavirus (HPV) 

Pelvic Inflammatory Disease 

Pelvic Tuberculosis 

Toxic Shock Syndrome (TSS) 

Chorioamnionitis 

HIV Transmission in Pregnancy 

Osteomyelitis 

Septic Arthritis — Gonococcal 



EH 

E3 



EH 



EE 



ACKNOWLEDGMENTS 



Throughoui the production of this book, we have had the sup- 
port of many friends and colleagues. Special thanks to our sup- 
port team including Anu Gupta, Andrea Fellows, Anastasia 
Anderson, Srishti Gupta, Mona Pall, Jonathan Kirsch and Chirag 
Ainin. For prior contributions we thank Gianni Le Nguyen, 
Tarun Mathur, Alex Grimm, Sonia Santos and Elizabeth 
Sanders. 

We have enjoyed working with a world-class international pub- 
lishing group at Blackwell Science, including Laura DeYoung, 
Amy Nuttbrock. Lisa Flanagan. Shawn Girsberger, Lorna Hind 
and Gordon Tibbitts. For help with securing images for the 
entire series we also thank Lee Martin, Kristopher Jones, Tina 
Panizzi and Peter Anderson at the University of Alabama, the 
Armed Forces Institute of Pathology; and many of our fellow 
Blackwell Science authors. 

For submitting comments, corrections, editing, proofreading, 
and assistance across all of the vignette tides in all editions, we 

collectively thank: 

Tara Adamovich, Carolyn Alexander. Kris Alden, Henry E. 
Aryan, Lynman Bacolor, Natalie Barleneva, Dean Bartholomew, 
Debashish Behera, Sumit Bhatia, Sanjay Bindra, Dave Brinton, 
Julianne Brown, Alexander Brownie, Tamara Callahan, David 
Canes, Bryan Casey, Aaron Caughey, Hebert Chen.Jonadian 
Cheng, Arnold Cheung, Arnold Chin. Simion Chiosea, Yoon Cho, 
Samuel Chung, Gretchen Conant. Vladimir Coric, Christopher 
Cosgrove, Ronald Cowan, Karekin R. Cunningham, A. Sean 
Dalit y. Rama Dandamudi, Sunil Das, Ryan Armando Dave. John 
David, Emmanuel de la Cruz, Robert DeMello, Navneel Dhillon. 
Sharmila Dissanaike, David Donson, Adolf Etchegaray, Alea 
Eusebio, Priscilla A. Frase, David Frenz, Kristin Gaumer, 
Yohannes Gebreegziabher, Anil Gehi, Tony George, L.M. 
Gotanco, Parul Goyal, Alex Grimm, Rajeev Gupta, Ahmad 
Halim. Sue Hall. David Hasselbacher, Tamra Heimert, Michelle 
Higley, Dan Hoit, Ericjackson, Tim Jackson, Sundarjayaraman, 
Pei-Nijone, Aarchan Joshi. Rajni K. Julia, Faiyaz Rapadi, Seth 
Karp, Aaron S. Kesselheim. Sana Khan, Andrew Pin-wei Ko, 
Francis Kong, Paul Konitzky, Warren S. Krackov. Benjamin H.S. 
Lau, Ann LaCasce, Connie Lee, Scott Lee, GuiUermo Lehmann, 
Kevin Leung, Paul Levett, Warren Levinson. Eric Ley, Ken Lin, 






Pavel Lobanov, J. Mark Maddox, Aram Mardian, Samir Mehta, 
Gil Melmed, Joe Messina. Robert Mosca, Michael Murphy, Vivek 
Nandkarni, Siva Naraynan, Carvell Nguyen, Linh Nguyen, 
Dcanna Nobleza, Craig Nodurft, George Noumi, Darin T. 
Okuda. Adam L. Palance, Paul Pamphrus, Jinha Park, Sonny 
Palel, Ricardo Pietrobon, Riva L. Rahl. Aashiia Randeria, 
Radian Reddy, Beatriu Reig, Marilou Reyes, Jeremy Richmon, 
Tai Roe, Rick Roller. Rajiv Roy, Diego Ruiz, Andiony Russell, 
Sanjay Sahgal, Urmimala Sarkar,John Schilling, Isabell Schmitt, 
Daren Schuhmachcr, Sonal Shah, Fadi Abu Shahin, Mae Sheikh- 
Mi, Kdie Shen. Justin Smith. John Stulak. Lillian Su, Julie 
Sundaram, Rita Suri, Seth Sweetser, Antonio Talayero, Merita 
Tan. Mark Tanaka, Eric Taylor, Jess Thompson, Indi Trehan, 
Raymond Turner, Okafo L'chenna, Eric Uyguanco, Richa 
Vai ma, John Wages, Alan Wang, Eunice Wang. Andv Weiss, Amy 
Williams. Brian Yang, Hairy Zaky, Ashraf Zanian and David Zipf. 

For generously contributing images to the entire Underground 
Clinical Vignette Step 1 series, we collectively thank the staff at 
BlackweU Science in Oxford. Boston, and Berlin as well as: 

• Axford, J. Medicine. Osney Mead: BlackweU Science Ltd, 1996. 
Figures 2.14, 2.15. 2.16. 2.27, 2.28. 2.31. 2.35, 2.36. 2.38. 2.43, 
2.65a, 2.65b, 2.65c, 2.103b, 2.105b, 3.20b, 3.21, 8.27, 8.27b, 
8.77b, 8.77c, 10.81b, 10.96a, 12.28a, 14.6, 14.16, 14.50. 

• Bannister B. Begg N. Gillespie S. Infectious Disease, 2'"' Edition. 
Osney Mead: BlackweU Science Ltd, 2000. Figures 2.8, 3.4, 
5.28, 18.10, W5.32, W5.6. 

• Berg D. Advanced Clinical Skills and Physical Diagnosis. 
BlackweU Science Ltd., 1999. Figures 7.10, 7.12. 7.13, 7.2. 7.3. 
7.7.7.8. 7.9,8.1.8.2.8.4,8.5,9.2, 10.2. 11.3. 11.5, 12.6. 

• Cuschieri A, Hennessy TPJ, Greenhalgh RM. Rowley DA, 
Grace PA. Clinical Surgery. Osney Mead: BlackweU Science 
Ltd, 1996. Figures 13.19. 18.22, 18.33. 

• Gillespie SH, Bamford K. Medical Micmbiology and Infection at a 
Crlance. Osney Mead: BlackweU Science Ltd, 2000. Figures 20, 23. 

• Ginsberg L. Lecture Notes on Neurology, 7"' Edition. Osney Mead: 
BlackweU Science Ltd. 1999. Figures 12.3, 18.3, 18.3b. 



• 



Elliott T. Hastings M, Desselbetgei l\ Lecture \ote\ on Medical 
Microbiology, 3"' Edition. Osney Mead: BlackweU Science Ltd, 
1997. Figures 2, 5, 7, 8, 9, 11, 12, 14, 15, 16, 17, 19, 20, 25, 26, 
27, 29, 30, 34, 35, 52. 



• Mehta AB, Hoffbrand AV. Ihii>matolog\ nl n (llanre. Osney 
Mead: Blackwell Science Ltd, 2000. Figures 22.1, 22.2, 22.3. 

Please lei us know if your name has been missed or misspelled 
and we will be happy to make the update in the next edition. 






PREFACE TO THE 3RD EDITION 



We were very pleased with the overwhelmingly positive student 
feedback for the 2nd edition of our Underground Clinical Vignettes 
series. Well over 100,000 copies of the UCV books are in print 
and have been used by students all over the world. 

Over the last two years we have accumulated and incorporated 
over a thousand "updates" and improvements suggested by you, 
our readers, including: 

• many additions of specific boards and wards testable content 

• deletions of redundant and overlapping cases 

• reordering and reorganization of all cases in both series 

• a new master index by case name in each Atlas 

• correction of a few factual errors 

• diagnosis and treatment updates 

• addition of 5— 20 new cases in every book 

• and the addition of clinical exam photographs within UCV — 
Anatomy 

And most important of all, the third edition sets now include 
two brand new COLOR ATLAS supplements, one for each 
Clinical Vignette series. 

• The UCV-Basic Science Color Atlas (Step 1) includes over 250 
color plates, divided into gross pathology, microscopic pathol- 
ogy (histology), hematology, and microbiology (smears). 

• The I 'CV-Clinical Science Colin- Alias (Step 2) has over 125 color 
plates, including patient images, dermatology, and fundus- 
copy. 

Each atlas image is descriptively captioned and linked to its cor- 
responding Step 1 case. Step 2 case, and/or Step 2 MiniCase. 






How Atlas Links Work: 



Step 1 Book Codes are: A = Anatomy 



BS = Behavioral Science 

BC = Biochemistry 

M I = Microbiology, Vol. I 

M2 - Miciiiliicilnj>y. Vul. II 
PI = Pathophysiology, Vol. I 
P2 = Pathophysiology, Vol, II 
P:s = Pathophysiology;, Vol. Ill 

PH = PIkutji.h < llog) 



Step 2 Book Codes are: ER = Emergency Medicine 

IM1 = Internal Medicine. Vol. I 

IM2 = Internal Medicine. Vol. II 

NKI ' - Neurology 

OB = OB/GYN 

PEL) - Pediatrics 

SUR = Surgery 

PSY = Psychiatry 

MC = MiniCase 




Case Number 




I I I I M-P3-032A 



Indicates Type of Image: H = Hematology 
M = Microbiology 
PG = Gross Pathology 

I'M Microscopic Pathology 




ER-035A, ER-035B 



Indicates UCV1 or UCV2 Series 



• If" the Case number (032, 035, etc.) is not followed by a letter, 
then there is only one image. Otherwise A, B. C. D indicate 
up to 4 images. 

Bold Faced Links: In order to give you access to the largest 

number of images possible, we have chosen to cross link the 
Step I and 2 series. 

• If the link is bold-faced this indicates that the link is direct 
(i.e.. Siep 1 Case with the Basic Science Step 1 Atlas link). 

• If the link is not bold-faced this indicates that the link is indi- 
rect (Step 1 case with Clinical Science Step 2 Atlas link or vice 
versa) . 

We have also implemented a few structural changes upon your 
request: 

• Each current and future edition of our popular First Aid for 
the USAILE Step 1 (Appleton & Lange/McGraw-Hill) and First 
Mil for tlw USM1.F. Step 2 (Applelon & I.ange/ McGraw-Hill) 
book will be linked to the corresponding L'CV case. 

• We eliminated UCV — > First Aid links as they frequently 
become out of date, as the First Aid books are revised yearly. 



• The Color Atlas is also specially designed for quizzing — 
captions are descriptive and do not give away the case name 
direcdy. 

We hope the updated UCV series will remain a unique and well- 
integrated study tool dial provides compact clinical correlations 
to basic science information. They are designed to be easy and 
fun (comparatively) to read, and helpful for both licensing 
exams and the wards. 

We invite your corrections and suggestions for the fourth edi- 
tion of these books. For the first submission of each factual cor- 
rection or new vignette that is selected for inclusion in the 
fourth edition, you will receive a personal acknowledgment in 
the revised book. If you submit over 20 high-quality corrections, 
additions or new vignettes we will also consider inviting you to 
become a "Contributor" on the book of your choice. If you are 
interested in becoming a potential "Contributor" or "Author" 
on a future UCV book, or working with our team in developing 
additional books, please also e-mail us your CV/ resume. 

We prefer that you submit corrections or suggestions via 
elect tonic mail to UCVteam@yahoo.com. Please include 
"Underground Vignettes" as the subject of your message. If you 
do not have access to e-mail, use the following mailing address: 
Rlackwell Publishing. Ann: UCV Editors. 350 Main Street, 
Maiden, MA 02 148. USA. 

Vikas Bhushan 

Yishal Pall 
Too Le 
October 2001 






HOW TO USE THIS BOOK 



This series was originally developed to address the increasing 
number of clinical vignette questions on medical examinations, 
including the USMLE Step 1 and Step 2. It is also designed to 
supplement and complement the popular First Aid for the 
USMLE Step 1 (Appleton & Lange/McGravv Hill) and First Aid 
for the USMLE Step 2 (Appleton & Lange/McGraw Hill). 

Each UCV 1 book uses a series of approximately 100 "supra- 
prototypical" cases as a way to condense testable facts and 
associations. The clinical vignettes in this series are designed to 
incorporate as many testable facts as possible into a cohesive 
and memorable clinical picture. The vignettes represent 
composites drawn from general and specialty textbooks, 
reference books, thousands of USMLE style questions and the 
personal experience of the authors and reviewers. 

Although each case tends to present all the signs, symptoms, 
and diagnostic findings for a particular illness, patients gener- 
ally will not present with such a "complete" picture either clini- 
cally or on a medical examination. Cases are not meant to 
simulate a potential real patient or an exam vignette. All the 
boldfaced "buzzwords" are for learning purposes and are not 
necessarily expected to be found in any one patient with the 
disease. 

Definitions of selected important terms are placed within the 
vigneties in (smali < ys) in parentheses. Other parenthetical 
remarks often refer to the pathophysiology or mechanism of 
disease. The format should also help students learn to present 
cases succinctly during oral "bullet" presentations on clinical 
rotations. The cases are meant to serve as a condensed review, 
not as a primary reference. The information provided in this 
book has been prepared with a great deal of thought and care- 
ful research. This book should not, however, be considered as 
your sole source of information. Corrections, suggestions and 
submissions of new cases are encouraged and will be acknowl- 
edged and incorporated when appropriate in future editions. 









ABBREVIATIONS 




5-ASA 


5-aminosalicylic acid 


ABGs 


arterial blood gases 


ABVD 


adriamycin/bleomycin/vincrisUne/dacarbazine 


ACE 


angiotensin-converting enzyme 


ACTH 


adrenocorticotropic hormone 


AD II 


antidiuretic hormone 


AFP 


alpha fetal protein 


AI 


aortic insufficiency 


AIDS 


acquired immunodeficiency syndrome 


ALL 


acute lymphocytic leukemia 


ALT 


alanine transaminase 


AML 


acute myelogenous leukemia 


ANA 


antinuclear antibody 


ARDS 


adult respiratory distress syndrome 


ASD 


atrial septal defect 


ASO 


anti-streptolysin O 


AST 


aspartate transaminase 


AV 


arteriovenous 


BE 


barium enema 


BP 


blood pressure 


BUN 


blood urea notrogen 


GAD 


coronary artery disease 


CALLA 


common acute lymphoblastic leukemia antigen 


CBC 


complete blood count 


CHF 


congestive heart failure 


CK 


creatine kinase 


CLL 


chronic lymphocytic leukemia 


CML 


chronic myelogenous leukemia 


CMV 


cytomegalovirus 


CNS 


central nervous system 


COPD 


chronic obstructive pulmonary disease 


CPK 


creatine phosphokinase 


CSF 


cerebrospinal fluid 


GT 


computed tomography 


CVA 


cerebrovascular accident 


CXR 


chest x-ray 


DIG 


disseminated intravascular coagulation 


DIP 


distal interphalangeal 


DKA 


diabetic ketoacidosis 


DM 


diabetes mellitus 


DTRs 


deep tendon reflexes 


DVT 


deep venous thrombosis 






EBV Epstein-Barr virus 

ECG electrocardiography 

Echo echocardiography 

11- ejection fraction 

EGD esophagogastroduodenoscopy 

EMG electromyography 

ERCP endoscopic retrograde cholangiopancreatography 

ESR erythrocyte sedimentation rate 

FEV forced expiratory volume 

FNA fine needle aspiration 

FTA-ABS lluorescent treponemal antibody absorption 

FVC forced vital capacity 

GFR glomerular filtration rate 

GH growth hormone 

(■I gastrointestinal 

GM-CSF granulocyte macrophage colony stimulating 

factor 

GU genitourinary 

HAY hepatitis A virus 

hcG human chorionic gonadotropin 

HEENT head, eyes, ears, nose, and throat 

HIV human immunodeficiency virus 

HLA human leukocyte antigen 

HPI history of present illness 

HR heart rate 

HRIG human rabies immune globulin 

HS hereditary spherocytosis 

ID/CC identification and chief complaint 

1DDM insulin-dependent diabetes mellitus 

Ig immunoglobulin 

IGF insulin-like growth factor 

IM intramuscular 

JVP jugular venous pressure 

KUB kidneys/ureter/bladder 

LDH lactate dehydrogenase 

LES lower esophageal sphincter 

I I I s liver function tests 

LP lumbar puncture 

LV left ventricular 

I,VH left ventricular hypertrophy 

Lytes electrolytes 

MCHC mean corpuscular hemoglobin concentration 

MCV mean corpuscular volume 

MEN multiple endocrine neoplasia 
















MGUS 


monoclonal gammopathy of undetermined 








significance 






MHC 


major histocompaiibility complex 






MI 


myocardial infarction 






MOPP 


mechloreihamine/vincristine (Oncovorin)/ 






procarbazine/prednisone 






MR 


magnetic resonance (imaging) 






MIL. 


non-Hodgkin's lymphoma 






NIDDM 


non-insulin-dependent diabetes mellitus 






NPO 


nil per os (nothing by mouth) 






NSAID 


nonsteroidal anti-inflammatory drug 






PA 


posteroanterior 






PIP 


proximal interphalangeal 






PBS 


peripheral blood smear 






PE 


physical exam 






PFTs 


pulmonary function tests 






PMI 


point of maximal intensity 






PMN 


polymorphonuclear leukocyte 






PT 


prothrombin time 






PTCA 


percutaneous transluminal angioplasty 






PTH 


parathyroid hormone 






pit 


partial thromboplastin time 






PUD 


peptic ulcer disease 






RBC 


red blood cell 






RPR 


rapid plasma reagin 






RR 


respiratory rale 






RS 


Reed-Stern berg (cell) 






RV 


right ventricular 






RVH 


right ventricular hypertrophy 






SBFT 


small bowel follow-through 






S1ADH 


syndrome of inappropriate secretion of ADH 






SLE 


systemic lupus erythematosus 






STD 


sexually transmitted disease 






TFTs 


thyroid function tests 






tPA 


tissue plasminogen activator 






TSH 


thyroid-stimulating hormone 






TIBC 


total iron-binding capacity 






TIPS 


uansjiigular intrahepatic portosystemic shunt 






TPO 


thyroid peroxidase 






TSH 


thyroid-stimulating hormone 






TTP 


thromboiic thrombocytopenic purpura 






I'A 


urinalysis 






UGI 


upper C.l 






US 


ultrasound 













VDRL Venereal Disease Research Laboratory 

VS vital signs 

VT ventricular tachycardia 

WBC white blood cell 

WPW Wolff-Parkinson-White (syndrome) 

XR x-ray 



ID/CC A 30-year-old female presents to the KR with severe, sudden- 
onset shortness of breath and an extensive pruritic skin rash. 

HPI She was prescribed cotrimoxazole by her general physician for a 
LI I: she took die first close only a few minutes before developing 
symptoms. 

PE VS: hypotension. PE: severe respiratory distress; central cyanosis; 
extensive urticaria] wheals noted all over body. 

Labs IgE antibody demonstrated to sulfonamides by RAST. 

Treatment Epinephrine (1:1000); antihistaminics; steroids; ventilatory 

support; adequate IV fluid administration or vasopressor agents 
to treat hypotension. 

Discussion Systemic anaphylaxis is the most serious and life-threatening 

IgE-mediated type I hypersensitivity reaction; its recognition and 
prompt treatment are critical to survival. 



o 

o 
en 

-< 



ANAPHYLAXIS 









ID/CC A 2-year-old male is admitted to the hospital for evaluation of a 
suspected immune disorder. 

HPI He has a history of recurrent fungal diaper rashes and 

staphylococcal cervical furunculosis requiring multiple incisions 
and drainage in addidon to antibiotics. His mother also reports 
chronic diarrhea and a prior perianal fistula. 

PE Cervical lymphadenopathy: mild hepatomegaly and 
splenomegaly; no pallor, purpuric patches, or sternal 
tenderness. 

Labs CBC/PBS: neutrophilic leukocytosis. Elevated ESR; normal 

serum immunoglobulins; absence of respiratory burst (negative 
nitroblue tetrazolium lest and chemoluminescence assay); 
negative Mantoux test. 



Imaging 



Micro Pathology 



CXR: hilar lymphadenopathy. US. abdomen: hepato- 
splenomegaly; hepatic and splenic nodular lesions (due to 
granulomas). 

Characteristic granuloma formaiion with phagocytes, giant cells, 
and occasional histiocytes in lymph nodes, liver, spleen, and 
lungs. 



Treatment Long-term TMP-SMX prophylaxis, -y-interferon. 

Discussion Chronic granulomatous disease is most commonly an X-linked 
disorder of ntuirophil function (may have variable inheritance 
patterns) that is due to a deficiency of NADPH oxidase. 
Neutrophils of affected padents demonstrate normal 
chemotaxis, degranulation. and phagocytosis but cannot use the 
oxygen-dependent myeloperoxidase system for microbial killing, 
making patients susceptible to recurrent staphylococcal infections. 



CHRONIC GRANULOMATOUS DISEASE 



ID/CC A 19-year-old male has recurrent attacks of bilateral periorbital 
and hand swelling coupled with respiratory difficulty that lasts 
up to 24 hours and often requires hospitalization. 

HPI He does not, however, complain of itching. His father and his 
aunt both suffer from a similar illness. 

PE Physical examination unremarkable. 

Labs Decreased C4 (besi screening test); decreased CI inhibitor 

(confirmatory test) and C2; normal C3; normal IgE. 

Treatment Synthetic androgens (e.g., dana/ol), fresh frozen plasma. 

Discussion CI esterase inhibitor deficiency is inherited as an autosomal- 
dominant u ail; death may result from laryngeal edema. Also 
known as hereditary angioedema. 



O 

en 

-< 



HEREDITARY ANGIOEDEMA 



ID/CC A 25-year-old white female is referred to an internist by her 

family doctor for a workup of recurrent sinusitis, chronic otitis 
media, one episode of pneumonia that required hospitalization, 
and recurrent bouts of watery diarrhea. 

HPI She has seen an allergy specialist for several years and has 

received desensitization shots for multiple allergies, including 
pollen, dust, and cat hair. 

PE Normal except for hypopigmented spots on neck and arms 

(VITILIGO). 

Labs Markedly decreased serum IgA; normal IgG and IgM. 

Imaging XR, sinus: opacification of paranasal sinuses (due to chronic 
sinusitis). 

Treatment Largely supportive; antibiotic therapy; try to avoid blood or plasma 
transfusion (anaphylaxis or serum sickness due to presence of 
antibodies to IgA) . 

Discussion Selected IgA deficiency is the most common congenital 

immunodeficiency, especially in patients of European descent. 
Diarrhea is usually caused by Giardia lambda; recurrent 
sinopulmonary infections are caused by Streptococcus pneumoniae, 
Haemophilus influenzae, or Staphylococcus aureus; associated with 
an increased incidence of allergies and autoimmune diseases 
such as SLE and rheumatoid arthritis. Selective IgA deficiency 
may be due to a specific defect in isotype switching. 



SELECTIVE IgA DEFICIENCY 



" 



ID/CC A 4-month-old male presents with chronic diarrhea and failure 
to thrive. 

HPI The infant was diagnosed with extensive mucocutaneous 

candidiasis in the early neonatal period and shortly thereafter 
developed a fulminant Pseudomonas septicemia that required 
intravenous antibiotic therapy for an extended period of time. A 
paternal cousin had developed similar and equally devastating 
bacterial and fungal infections in the neonatal period and 
subsequently died. 

PE Emaciated; mucocutaneous candidiasis noted; tonsils not seen; 
lymph nodes not palpable despite recurrent infections. 

Labs CBC: severe lymphopenia. PBS: lack of mature lymphocytes. 
Tests for cutaneous delayed hypersensitivity and contact 
sensitization negative; serum immunoglobulin levels (IgG, IgA, 
and IgM) low; adenosine deaminase (ADA) deficiency demon- 
strated in red cells. 

Imaging ( 1XR: absent thymic shadow. 



Gross Pathology 



Micro Pathology 



Treatment 



Discussion 



Thymus fails to descend into the anterior mediastinum from the 
neck and resembles fetal thymus of 6 to 8 weeks. 

No lymphoid tissue in the lymph nodes, spleen, tonsils, and 
appendix. 

Bone marrow transplant from an HLA-identical sibling; IV 
immunoglobulin; infusion of normal ADA-containing erythro- 
cytes (ADA-PEG is also very successful); antibiotics; gene therapy 
for ADA; genetic counseling (SCID caused by ADA deficiency 
can be diagnosed prenatally by amniocentesis) . 

Severe combined immunodeficiency syndrome is characterized 
by marked depletion of the cells that mediate bodi humoral 
(B-cell) and cellular (T-cell) immunity. SCID may be transmitted 
as either an autosomal-recessive trait or an X-linked recessive 
trait, or it may be sporadic; half of the cases inherited in an 
autosomal-recessive manner are caused by a deficiency in ADA. 






o 

-< 



SEVERE COMBINED IMMUNODEFICIENCY (SCID) 






ID/CC A 7-month-old male is admitted for a workup of recurrent upper 
respiratory tract and skin infections of several months' duration. 

HPI His parents state that he has had recurrent URIs, one episode of 
Haemophilus influenzae pneumonia, and severe odtis media. 

PE Low weight and height for chronological age; chronic bilateral 
suppurative otitis media; asymmetric arthritis of knees; no tonsillar 
tissue seen; no lymphadenopathy or hepatosplenomegaly. 

Labi Panhypogamrnaglobulinernia: very low IgG; IgA and IgM 
undetectable. 

Treatment Parenteral gamma globulin; monitor pulmonary function to 
guard against chronic lung disease. 

Discussion An X-linked disease (manifests only in males) characterized by a 
selective B-cell defect with recurrent bacterial infections. Also 
known as Bruton's disease, X-linked hypogammaglobulinemia is 
due to a genetic defect in tyrosine kinase receptor found on 
antibody precursors, resulting in impaired maturation and 
development of antibodies. Male infants demonstrate infections 
when maternal antibodies have cleared from their system. 



X-LINKED HYPOGAMMAGLOBULINEMIA 






ID/CC A 27-year-old white female complains of mouth ulcers, 

prolonged fever, flulike symptoms, and increasing fatigue and 
weight loss over the past 2 months. 

HPI She recently moved from a large metropolitan area to a farm in 
Ohio, where she spent 1 week cleaning a pigeons' loft. 

PE VS: fever (38.5°C). PE: pallor; weight loss; enlarged liver and 
spleen; generalized lymphadenopathy: sc altered sibilanl rales 
over lung fields. 



Labs 



Imaging 



Gross Pathology 



Micro Pathology 



( !'>( :/PBS: anemia; leukopenia. Small, budding fungus found 
intracellularly in reticuloendothelial cells (macrophages) on silver 
stain; elevated l.DH; positive blood culture for dimorphic 
fungus. 

CXR: nonsegmental shifting pneumonic infiltrates; mediastinal 
adenopathy with popcorn calcifications; bilateral hilar adenopa- 
thy. CT, abdomen: splenic calcifications. 

Nodules with granuloma formation; central area of necrosis and 
caseation with sclerosis and calcification; any organ may be in- 
volved, mainly reticuloendothelial system (RES) and adrenals. 

Granulomas with epithelioid cells, Eanghans' giant cells, and 
organisms within macrophages; in disseminated disease, 
organisms present in RES throughout body with proliferation. 



Treatment Itraconazole; amphotericin B. 

Discussion Histoplasmosis is a systemic fungal infection sometimes resembling 
TB that is caused by Hislojilasiiia ai/i.\ulatam, a dimorphic fungus. 
The \east form is found intracellularly: the mold form is found 
in soil associated with bird or bat feces. Transmitted by inhalation 
of mold spores, it varies in intensity from asymptomatic to fulmi- 
nant (in immunocompromised patients). The disease is most 
prevalent in the southeastern, mid-Atlantic, and central regions 
of the United States. 



o 

— I 
I— I 
O 

a 

o 
i — i 
oo 

m 
> 



HISTOPLASMOSIS 



ID/CC A 57-year-old black male complains to his doctor of increasing 
weakness, swollen glands in the armpits and groin, and a feeling 
of heaviness in the abdomen (due to hepatosplenomegaly) . 

HPI The patient is an immigrant from Trinidad and Tobago and has 
a history of nonresolving skin rashes and recurrent respiratory 
infections. 

PE Marked pallor; extensive papular skin rash with few erythematous 
plaques over abdomen; generalized lymphadenopathy and 
hepatosplenomegaly. 

Labs CBC/PBS: marked leukocytosis (83,000) with relative lymphocy- 
tosis and atypical lymphocytes. Increased LDH; hypercalcemia. 

Imaging CXR: normal. 

Micro Pathology Skin biopsy reveals infiltration by leukemic CD4+ T lymphocytes. 

Treatment Aggressive combination chemotherapy. 

Discussion Adult T-cell leukemia/lymphoma (ATLL) is associated with 
HTLV-1 type C, a retrovirus that has a higher incidence in 
blacks from the Caribbean and southeastern United States as 
well as in people from southern Japan and sub-Saharan Africa. 
The infection is acquired via transmission from mother to child 
(breast milk), from sexual activity, from blood transfusion, or 
from IV drug use. 



HUMAN T-CELL LEUKEMIA VIRUS TYPE 1 (HTLV-1) 






ID/CC A 2-day-old neonate is evaluated for an eye discharge. 

HPI The baby's mother is a prostitute who did not receive any 
prenatal cervical cultures during pregnancy. 

PE Normal Ml-term male neonate; mucoid eye discharge, conjunctival 
congesdon, and chemosis noted in both eyes; nonfollicles seen 
on palpebral conjunctiva (due to absence of subconjunctival 
adenoid layer at this age); mild superficial keradds also present. 



Labs Gram stain of swab reveals increased PMNs and no bacteria; 

characteristic intracellular inclusion bodies demonstrated by the 
DIF test; cell culture yields Chlamydia trachomatis serotypes D 
through K; chlamydia also grown from maternal cervical swab. 

Treatment Erythromycin syrup; azithromycin suspension has also been 
shown to be beneficial; no topical therapy. 

Discussion Chlamydia trachomatis is an important cause of preventable 
blindness; its strains can be further differentiated into 18 
serotypes by mieroimmunofluorescence tests. Serotypes A, B, 
Ba, and C are principally associated with endemic trachoma in 
developing countries; serotypes D through K primarily cause 
sexually transmitted infections in adults and inclusion 
conjunctivitis and pneumonia in infants, transmitted through an 
infected birth canal; and serotypes LI, L2, and L3 cause 
lymphogranuloma venereum. 



m 
r-> 

—I 

I— I 
O 

cz 
on 

o 

i— i 
on 

m 
> 



INCLUSION CONJUNCTIVITIS 



ID/CC 



HPI 



A 20-year-old male college student complains of sore throat, 
fatigue, fever, swollen lymph nodes on the back of his neck, 

anorexia, cough, and malaise of 10 days' duration. 

He was initially given ampicillin by his school nurse, after which 
he developed an extensive skin rash. 



PE VS: fever. PE: enlargement of submaxillary and cervical lymph 
nodes; exudadve tonsillitis: petechiae on soft palate; slightly- 
enlarged spleen and liver. 



Labs 



Gross Pathology 



Micro Pathology 



CBC/PBS: anemia; thrombocytopenia; leukocytosis with absolute 
lymphocytosis (50%); atypical lymphocytes. Elevated ALT, AST, 
and bilirubin; positive heterophil antibody test (Paui.-Bunni .i I. 
TEST); IgM antibodies to viral capsid andgen/monospot positive. 

Enlarged spleen, lymph nodes, and, to lesser extent, liver; 
hepatitis may be present along with brain involvement; splenic 
rupture rare complication. 

Proliferation of reticuloendothelial system; infiltration of spleen 
by atypical lymphocytes. 



Treatment Supportive. 



Discussion 



Infectious mononucleosis is a systemic viral infection that is 
caused by Epstein-Barr virus (EBV). a herpesvirus, and is 
transmitted through respiratory droplets and saliva. In developed 
countries, it most commonly affects teenagers and young adults 
("kissing disease"); in underdeveloped countries, it is seen as a 
subclinical infection of early childhood. EBV infection is associ- 
ated with an increased risk of Burkitt's lymphoma, Hodgkin's 
disease, and nasopharyngeal carcinoma. 



Atlas Links 



H-M2-010 



...J f. ! IM2-022 



J3 INFECTIOUS MONONUCLEOSIS 






ID/CC A 65-year-old male presents with a high fever, headache, extreme 
prostration, a nonproductive cough, and severe breathlessness. 

HPI He had been receiving chlorambucil for treatment of chronic 
lymphocytic leukemia (CLL) and was in an extremely 
debilitated state. 

PE VS: fever; tachypnea; cyanosis. PE: conjunctival congestion; 

pharyngeal inflammation; rales and wheezes heard on auscultation 
over both lung fields; splenomegaly and lymphadenopathy (due 
to CLL). 



Labs 



Imaging 



Treatment 



Discussion 



No organisms seen or cultured from sputum; fluorescent 
antibody directed against influenza virus was positive; viral 
cultures of nasopharyngeal washings grew influenza virus; 
fourfold rise in hemagglutination inhibition antibody titer 
against influenza virus demonstrated. 

CXR (PA view): bilateral, diffuse interstitial infiltrates suggestive 
of atypical pneumonia. 

Amantidine or rimantadine for influenza A (zanamivir or 
oseltamivir for influenza A and B); ventilatory support, antipyretics, 
and IV fluids. Secondary staphylococcal pneumonia should be 
treated with parenteral antibiotics; yearly vaccination prevents 
excessive morbidity and mortality, especially among the elderly. 

Influenza viruses are medium-sized spherical RNA viruses 
termed orthomyxoviruses; influenza A and B viruses each con- 
tain 8 RNA segments and ID viral proteins. Influenza infection is 
most common in winter, with the severity of a given outbreak 
depending on the status of immunity in the community. 
Previous natural infection or immunization with viruses that are 
immunologically dose to the current strain limits new infection, 
bin if antigenic drift results in reduced cross-reactivity, the new 
strain will spread more rapidly. New strains produced by 
antigenic shift account for most major outbreaks. Influenza 
affects all segments of the population, but severe infections and 
major complications are most common in patients who are 
young, elderly, or debilitated. 




INFLUENZA 






ID/CC A 30-year-old female presents with fever, chills, malaise, 
headaches, and myalgias. 

HPI She was diagnosed as suffering from secondary syphilis with an 
extensive nonpruritic skin rash, condylomata lata, and mucous 
patches in the mouth, for which she received a dose of 
intramuscular penicillin 6 hours ago. 

PE VS: fever; tachycardia; mild hypotension. 

Treatment No specific treatment; symptoms subside in 24 hours. 

Discussion The Jarisch-Herxheimer reaction consists of fever, chills, mild 
hypotension, headache, and an increase in the intensity of 
mucocutaneous lesions 2 hours after initiating treatment of 
syphilis with penicillin or another effective antibiotic; symptoms 
usually subside in 12 to 24 hours. The reaction occurs in 50% of 
patients with primar\ syphilis .mil in 90% oJ [hose with secondary 
syphilis. The Jarisch-Herxheimer reaction also occurs after 
treatment of other spirochetal diseases (e.g., louse-borne relapsing 
lever caused b\ Borrelia recurrentis) . It has been suggested that 
the release of treponemal lipopolysaccharides might produce 
this symptom complex. 



JARISCH-HERXHEIMER REACTION 



ID/CC A 40-year-old male smoker complains of acute-onset high fever, 
chills, a nonproductive cough, tachypnea, and pleuritic chest 
pain. 

HPI A number of similar cases have been reported in his workplace 
in recent months. The patient admits to significant alcohol and 
tobacco consumption and uses a humidifier at night. 



Pt 



Labs 



Imaging 



VS: fever; dyspnea. PE: rales present bilaterally on auscultation. 

Sputum exam with Gram stain reveals no pathogenic organisms. 
CBC: neutrophilic leukocytosis. Cold agglutinins absent; indirect 
fluorescent antibody technique reveals stable titer of > 1:256 
(considered diagnostic); direct immunofluorescent staining of 
sputum confirms presence of Legionella. 

CXR, PA: bilateral diffuse, patchy infiltrates and ill-defined 
nodules. 



Gross Pathology Nodular areas of consolidation that may progress to involvement 
of one or more lobes of the lung. 

Micro Pathology Alveolar exudate with PMNs, macrophages, and fibrin; in more 
severe cases, destruction of alveolar septa. 

Treatment Erythromycin or an active fluoroquinolone. 

Discussion Legionnaire's disease is caused by Legionella pneumophila, a 

filamentous, flagellated, aerobic gram-negative, motile bacillus, 
and is more common in immunocompromised patients. 
Epidemiologic studies have established drinking water and air 
conditioners as the sources of outbreak. 



— i 
i — i 
o 
c: 
oo 

o 

HH 

LO 

m 
> 



LEGIONELLA PNEUMONIA 



ID/CC A 30-year-old Pakistani immigrant complains of chronic fever, 
weight loss, increased abdominal girth, a feeling of heaviness, 
and appetite loss. 

HPI Almost a year ago, the patient had a small, pruritic red papule on 
his left arm that was caused by an insect bite and disappeared 
spontaneously. 

PE Skin darkening; trophic changes in hair; massive nontender, 

hard splenomegaly; hepatomegaly without jaundice; generalized 
lyinphadenopathy: peripheral edema; ecchynaosis. 

Labs CBC/PBS: anemia, leukopenia, thrombocytopenia 

(pancytopenia), and monocytosis; amastigotes in buffy coat. 
Hypergammaglobulinemia; decreased albumin; increased ALT 
and AST. 

Imaging CT/US, abdomen: splenomegaly. 



Gross Pathology 



Micro Pathology 



Massively enlarged spleen; also greatly increased in weight, dark 
colored, and congested with Leishman-Donovan bodies. 

Proliferation of reticuloendothelial system cells; biopsy or 
aspiration reveals parasite-filled macrophages in infected locations. 



Treatment Pentavalent antimony (e.g., sodium stibogluconate): 

amphotericin B or pentamidine iscthionate. 

Discussion Also known as kala azar, leishmaniasis is a zoonosis that is produced 
by Ijoshmania donovani and is transmitted through the bite of 
the Phlebotomuh sandfly. It is associated with a high fatality rate 
when left untreated. 

Atlas Links Lih.U M-M2-014A, M-M2-014B, M-M2-014C, H-M2-014 



LEISHMANIASIS 






ID/CC A 30-year-old male from India presents with slowly progressive 
hypopigmented skin patches and nodules together with a 
peculiar deformity of the nose. 

HPI The patient has a history of nasal stuffiness and bloody nasal 
discharge; he also complains of loss of libido. 

PE Leonine facies (thickened facial and forehead skin); loss of 

eyebrows and eyelashes (MADAROSIS); scleral nodules; depressed 
nasal bridge ("SADDLE-NOSE* DEFORMITY); gynecomastia; testicular 
atrophy; numerous symmetrical, hypopigmented macules with 
vague edges and erythematous, smooth, shiny surfaces; skin 
plaques and nodules; partial loss of pinprick and temperature 
sensation (HYPOESTHESIA); no anhidrotic changes; symmetrically 
enlarged ulnar and common peroneal nerves. 

Labs CBC: mild anemia. ESR elevated; slil skin smears reveal 
numerous acid-fast bacilli on modified ZN staining. 



Micro Pathology 



Dermis massively and diffusely infiltrated with foamy histiocytes 
with bacilli and globi (masses of acid-fast bacilli) containing 
Vii chow giant cells; bacilli found only rarely in epidermis and in 
subepidermal "clear zone"; epidermis thinned out widi flatten- 
ing of rete ridges. 



Treatment Multidrug iherapy with rifampicin, dapsone, and clofazimine. 

Discussion The discovery of one or more of the following is pathogno- 
monic of leprosy: (1) anesthetic skin lesions (found in all tuber- 
culoid and many lepromatous cases): (2) thickening of one or 
more nerves (found in many lepromatous and some tuberculoid 
cases); and (3) the presence of acid-fast bacilli in skin smears 
(found in all lepromatous and some tuberculoid cases). 
Myrobiictrrium Irpmehas not been cultured in vitro thus far. 
Frequent complications include hand crippling (secondary to 
nerve damage) and blindness. It is currently believed dtat in 
most instances, the mode of transmission is via person-to-person 
contact. 

Atlas Links LU.CE3 :J MC-179A, MC-179B 



LEPROSY— LEPROMATOUS 



o 

—I 

I — I 

o 

IS) 

o 

I— I 

co 
m 

> 



ID/CC A 26-year-old male from India presents with a hypopigmented, 
anesthetic skin patch over the left side of his face. 

HPI He also complains of an occasional "electric current"-like 
sensation radiating from his left elbow to his hand. 

PE Dry, hypopigmented. anesthetic patch over left cheek; left ulnar 
nerve enlarged and palpable; eye, ear, nose, and throat exam 
normal; testes normal (vs. signs that are often demonstrable in 
lepromatous leprosy). 

Labs Glucose-6-phosphate dehydrogenase (G6PD) levels within 

normal range (done to prevent dapsone-associated hemolysis); 
slit skin smears reveal few acid-fast bacilli; skin biopsy from 
patch diagnostic of tuberculoid leprosy. 

Gross Pathology Single or small number of lesions with macular or raised edges. 



Micro Pathology 



Skin biopsy reveals many well-formed epithelioid granulomas 
with very few acid-fast bacilli. 



Treatment Chemotherapy with rifampin and dapsone. 

Discussion Caused by Mycobacterium leprae, an acid-fast bacillus. The orga- 
nism has two unique propenies: il is thermolabile. growing best 
31 27°C to 30°C, and it divides very slowly; generation time is 12 
to 14 days. Consequently, leprosy in humans typically evolves 
very slowly. Tuberculoid leprosy predominantly affects the skin 
with limited nerve involvement (most commonly ulnar - and per- 
oneal); lepromatous leprosy has diffuse involvement of the skin, 
eyes, nerves, and upper airway with disfigurement of the hands 
and face (leonine facies). 

Atlas Links mcmzi MC-179A, MC-179B 



LEPROSY— TUBERCULOID 



ID/CC A 35-year-old Britisli dairy farmer complains of a high remittent 
fever with chills, severe muscle aches, decreased urine output, 
and dark-colored urine for the past 2 days. 

HPI He also complains of an extensive skin rash and nasal bleeding 
(epistaxis). A careful history reveals that the area in which he 
works is infested with rodents. 

PE VS: fever; tachycardia; hypotension. PE: icterus; extensive hem- 
orrhagic maculopapular skin eruption; conjunctival suffusion; 
lymphadenopathy. 

Labs CBC: leukocytosis with neutrophilia; thrombocytopenia. Mild 
hyperbilirubinemia, predominantly conjugated; increased 
alkaline phosphatase; elevated BUN and creatinine. UA: 

proteinuria, casts, and RBC.s. Blood culture (positive during first 
10 days of illness) and urine culture (positive after second week 
of infection) on Fletcher's medium isolated Ixtptospira interrogans; 
serologic diagnosis (positive during second week of illness): 
microscopic slide agglutination demonstrated significant titer of 
antibody to /.. interrogans. 

Imaging CXR: patchy alveolar infiltrates consistent with alveolar 
hemorrhage. 

Gross Pathology Severe infection damages both the liver and kidneys. 



Micro Pathology 



Liver biopsy shows focal centrilobular necrosis with focal lympho- 
cytic infiltration and disorganization of liver cell plates together 
with proliferation of Kupffer cells with cholestasis; kidney biopsy 
reveals mesangial proliferation with PMN infiltration. 



Treatment Penicillin (dose modified due to presence of renal failure), 
doxycycline; hemodialysis. 

Discussion Weil's disease, a severe form of leptospirosis caused by LejHospira 
interrogans complex, is characterized by fever, jaundice, cuta- 
neous and visceral hemorrhages, anemia, azotemia, and altered 
consciousness; major vectors to humans are rodents. 
Transmission occurs through direct contact with the blood, tis- 
sue, or urine of infected animals. Person-to-person transmission 
is highly unlikely. Preventive measures include limiting the ro- 
dent population and vaccinating animals. 



o 

cz 
oo 

I — I 

m 

3> 



LEPTOSPIROSIS (WEIL'S DISEASE) 



ID/CC A 2-week-old female is brought to the emergency room because 
of high fever and convulsions. 

HPI She also has an extensive skin rash on her legs and trunk. 

PE VS: fever. PE: generalized hypotonia: extensive maculopapular 
skin rash; nuchal rigidity; involuntary flexion of hips when 
flexing neck (Brudzinski's sign). 

Labs CBC: neutrophilic leukocytosis. LP: elevated CSF cell count 

(750 cells/ mL), mostly neutrophils; elevated CSF protein; low 
CSF sugar. Cram-positive, facultative, intracellular, nonsporulat- 
ing motile bacilli on Cram stain and culture. 

Gross Pathology Purulent meningitis. 



Micro Pathology 



Bacillus provokes both acute suppurative reaction with neu- 
trophilic infiltration and chronic granuloma formation with 
focal necrosis. 



Treatment IV antibiotics (high-dose ampicillin). 

Discussion Listeriosis is caused by Listeria monocytogenes. Bacteiial infection 
may occur early (acquired in utero) or later (drinking contami- 
nated milk) in neonatal life. Maybe rapidly fatal if disseminated. 
Also occurs in adults immunocompromised by disease (e.g., 
renal disease or HIV). Escherichia coii and group B streptococcus 
are two other common causes of neonatal meningitis. 



LISTERIOSIS 









ID/CC A 12-year-old male presents with fatigue, fever, headache, 

fleeting joint pain, and a reddish rash on his trunk and left leg 
of 1 week's duration. 

HPI The patient is a native of Connecticut and attended a summer 
camp in the state's national park 2 weeks ago. He recalls having 
noticed a tick bite on his leg about 2 weeks ago. 

PE VS: fever. PE: red macule on site of bite that has grown circum- 
ferentially; acdve border and central clearing (ERYTHEMA 
CHRON1CUM MIGRANS); femoral lymphadenopadiy; mild neck 
stiffness; normal CNS exam. 

Labs Posidve IgM ELISA for Borrelia burgdorferi; diagnosis confirmed 
by Western blot assay. ECG: normal. LP: lymphocytic pleocytosis; 
increased proteins. B. burgdorferi grown on Noguchi medium. 



Gross Pathology 



Erythema chronicum migrans (ECM) is characteristic of Lyme 
disease; must be minimum of 5 cm in diameter for diagnosis to 
be made; center may desquamate, ulcerate, or necrose; satellite 
lesions sometimes seen; may spontaneously disappear with time. 



Treatment Doxycycline: amoxicillin; ceftriaxone. 

Discussion The most common disease transmitted by vectors in the United 
States, Lyme disease is caused by Borrelia Imrgdorferi,, a spiro- 
chete, and is transmitted through Ixodes species dck bites. Ticks 
acquire B. burgdorferi from deer mice, which are the natural 
reservoir. There are three recognized stages: stage 1 consists of 
ECM and constitutional symptoms; stage 2, cardiac or neuro- 
logic involvement; and stage 3, persistent migratory arthrids, 
synovitis, and atrophic patches on the distal extremities 

(At RODliRMATITIS CHK( >\'ICl'M ATROPHICANS) . 



O 

cz 
1/1 

o 

t— I 
C/1 

m 
> 



LYME DISEASE 






ID/CC A 57-year-old black female from Kenya complains of increasing 
weight and edema of the lower legs with difficult)' walking. 

HPI Over the years she has had episodes of fever with swelling of 
inguinal lymph nodes and itching. She has also had numerous 
attacks of malaria. 

PE Inguinal lymph nodes indurated and slightly increased in size; 
marked deformity in both legs with thickening of skin and 
greatly increased diameter; rubbery consistency. 



Labs 

Imaging 

Gross Pathology 

Micro Pathology 

Treatment 
Discussion 



PBS: several microfilariae; prominent eosinophilia. 

Lymphangiogram: partial lymphatic obstruction at iliac level. 

Presence of adult worms in lymphatics; marked fibrosis sur- 
rounding obstructed vessels. 

Granulomatous reaction with plasma cell and lymphocytic infil- 
tration; giant cell formation: intense fibroblastic hyperplasia. 

Ivermectin; diethylcarbamazine; surgery in advanced cases. 

Lymphatic filariasis is a chronic disease that is due to lymphatic 
obstruction and is caused by several types of filarial round- 
worms, mainly Wuchereria bavoo/li and Brugiri malayi; it is trans- 
mitted by female mosquito bites. Also known as elephantiasis. 



Atlas Link EOT '1 M-M2-020 



LYMPHATIC FILARIASIS 






ID/CC A 25-year-old male complains of swollen, tender masses in his 
groin and very painful genital ulcers of 1 week's duration. 

HPI The patient admits to having had unprotected sex with multiple- 
partners. 

PE Swollen, erythematous, tender inguinal nodes, usually bilateral, 
with draining sinuses (inguinal adenitis, buboks); multiple small 
genital lesions. 



Labs 



Gross Pathology 



Micro Pathology 



Treatment 



Discussion 



Inguinal node biopsy diagnostic; positive complement fixation 
test; positive immunofluorescence test. 

Primary lesion is ulcerated nodule; gives rise to inguinal bubo, 

an enlarged lymph node sometimes characterized by fistulous 
tract formation; balanitis, phimosis, and rectal involvement with 
stricture may also be present. 

Neutrophilic infiltration of primary lesion with areas of necrosis; 
lymphoid hyperplasia of lymph nodes with foci of macrophage 
accumulation; abscess formation with fibrosis. 

Doxycycline; tetracycline; azithromycin; erythromycin; TMP- 
SMX; ceftriaxone; ciprofloxacin. 

Lymphogranuloma venereum is an STD that is due to 
Chlamydia trachomatis (LI, L2, L3). Counseling should be 
given about other STDs (e.g., AIDS, syphilis, gonorrhea). 



o 

cr 
co 

o 
i — i 

U1 

m 

> 
on 



LYMPHOGRANULOMA VENEREUM 



ID/CC A 30-year-old missionary conies to the emergency room 
complaining of high fever, chills, severe headache, and 
confusion. 

HPI Upon returning from Africa 2 weeks ago. he began to feel 
weak and experienced backaches, pain behind the eyes, and 
sleepiness. 

PE VS: fever (39°C); tachycardia. PE: pallor; profuse sweating; mild 
splenomegaly without lymphadenopailiy. 

Labs CBC/PBS: anemia; thrombocytopenia; plasmodia in 
erythrocytes on thick peripheral blood smear. Slight 
hyperbilirubinemia and hypoglycemia. 

Gross Pathology Liver and spleen moderately enlarged and soft in consistency, 
with sequestration and hemolysis of erythrocytes and 
macrophages; hyperplasia of Kupffer cells; malarial pigment in 
spleen and liver; brain capillaries may show thromboses. 

Micro Pathology Hypertrophy of phagocytic system: ischemic necrosis 
surrounding occluded blood vessels in brain. 

Treatment Chloroquine; quinine for cerebral malaria; sulfadiazine- 
pyrimethamine, mefloquine, tetracycline for areas with 
chloroquine-resistant strains; primaquine for radical treatment. 

Discussion Malaria is transmitted by female Anopheles mosquitoes. 

Plasmodium falciparum may be lethal, producing cerebral 
malaria. Other types include P. vivax, P. ovale, and P. malarmr. 

Atlas Links ._.! .:_] H-M2-022A, H-M2-022B, H-M2-022C 



MALARIA 






ID/CC A 3-year-old female is brought to the emergency room with a 
high fever of 7 days' duration, accompanied by redness of die 
eyes, persistent dry cough, and coryza. 

HPI Her family doctor had treated her illness as a viral URI, but no 
improvement was seen. One day before her admission, her 
mother noticed a skin rash starring behind her ears and face 
that has now spread to her trunk and extremities. 

PE Pallor; injected conjunctiva; hyperemic throat; erythematous 
maculopapular rash on face, neck, trunk, and extremities; 
retroauricular lymphadenopaihy; bluish-gray spots surrounded 
by erythematous areola on buccal mucosa in region of first 
molar (KOPLIK'S SPOTS) . 

Labs CBC: leukopenia. 



Gross Pathology 



Micro Pathology 



Koplik's spots pathognomonic of measles; appearance presages 
rash by approximately 2 days; uniform lesions (vs. varicella). 

Lymphocytic dermal infiltration; multinucleated giant cells in 
reticuloendothelial system (WARTHIN-FINKELDEY CELLS) . 



Treatment No specific antiviral therapy available; treat complications. 



Discussion 



Atlas Links 



Also called rubeola; not to be confused with rubella. Measles is 
produced by a paramyxovirus and is transmitted by respiratory 
droplets; a live attenuated vaccine is available. Measles has an in- 
cubation period of 10 to 14 days. Sequelae include encephalitis, 
subacute sclerosing panencephalitis (SSPE), and giant cell 
pneumonia. 

PED-030A, PED-030B 



MEASLES 



o 

—I 

O 

cz 
oo 

o 

1—1 
oo 

m 

> 



ID/CC A 12-year-old white female is brought to the emergency room 
because of sudden fever with chills, severe headache, pain in 
the extremities and back, stiff neck, and generalized rash; she 
also fainted while in school. 

HPI She had been well until admission, with no relevant history. In 
the emergency room, she vomits bright red blood twice. 

PE VS: tachycardia; hypotension (BP 70/50). PE: altered senso- 
rium; pallor; moist, cold skin; nuchal rigidity and positive 
Kernig's sign; petechial rash all over body; minimal papilledema 
on funduscopic exam; no focal neurologic signs. 

Labs Hypoglycemia. Lytes: hyponatremia; hyperkalemia. CBC/PBS: 
thrombocytopenia; neutrophilic leukocytosis. l.F: CSF cloudy 
and under increased pressure; increased proteins; low sugar. 
Gram-negative diplococci (Neisseria meningitidis) seen within and 
outside WBCs on Gram stain; negative India ink and ZN stain; 
growth of meningococci later revealed on blood culture. 

Imaging CT, head: normal. CT, abdomen: bilateral adrenal hemorrhage. 

Gross Pathology Bilateral adrenal hemorrhagic necrosis; skin necrosis; pyogenic 
meningitis. 



Micro Pathology 



Meningeal hyperemia with abundant purulent exudate; 
diplococcus-containing PMNs; acute hemorrhagic necrosis 
of adrenal glands. 



Treatment Steroid replacement; IV fluids; dopamine; IV penicillin G; 
prophylactic rifampin for close contacts. 

Discussion Meningococcemia is a fulminant disease caused by several 
groups of Neisseria meningitidis; the cause of death is adrenal 
necrosis with vascular collapse. A meningococcal vaccine is 
available. Also known as Waterhouse-Friderichsen syndrome. 

Atlas Link mcra2i IM2-024 



MENINGOCOCCEMIA 






ID/CC A 24-year-old white female with insulin-dependent diabetes 
meilitus (IDDM) is hospitalized for ketoacidosis following a 
night out drinking: on (he fifth day she develops right 
periorbital swelling and a mucopurulent postnasal discharge 
that fails to respond to antibiotics. 

HPI She admits to irregular adherence to glucose control and 
insulin dosing. 

PE Right periorbital and paranasal edema; swelling of conjunctiva 
(( i ii mosi.s): exophthalmos; black ulceration of nasal mucosa; 
third cranial nerve (CN III) palsy. 

Labs Large, irregular, nonseptate hyphae branching at wide (> 90°) 
angles on nasal culture. 



Imaging 
Gross Pathology 

Micro Pathology 

Treatment 

Discussion 



XR, plain: opacification of paranasal sinuses. 

Necrotic destruction of paranasal sinuses and orbit with dissemi- 
nation to lung and brain. 

Purulent arteritis with thrombi composed of hyphae; inflamma- 
tion and necrosis with polymorphonuclear infiltrate. 

Maintain tighter glucose control; amphotericin B; surgical 
drainage. 

Mucormycosis is a phycomycosis produced by Mucor and 
Rhizopus molds: it should be suspected in cases of antibiotic- 
resistant sinusitis, especially in the presence of underlying 
diabetes, burns, lymphoma, or leukemia. 



o 



on 
o 

K 

m 

on 



Atlas Link LUCta I I M-M2-025 



MUCORMYCOSIS 






ID/CC A 6-year-old white male presents with fever, nausea, vomiting, 
swelling, and tenderness of the mandibular angle; he finds it 
difficult to talk, eat, or swallow. 

HPI Two of his classmates were diagnosed with mumps 2 weeks ago. 
There is no vaccination record. 

PE VS: fever. PE: outward and upward displacement of ear; obliter- 
ated mandibular hollow; orifice of Stensen's duct swollen and 
hyperemic; right testicle enlarged and painful. 



Labs 



Gross Pathology 



Micro Pathology 



Treatment 
Discussion 



MUMPS 






CBC: leukopenia with lymphocytosis. Hyper amylasemia; positive 
complement fixation antibodies. 

Parotid glands enlarged with areas of necrosis and mononuclear 
infiltrate; encephalitis, orchitis, oophoritis, meningitis, and 
pancreatitis may also be present. 

Examination of parotid glands reveals perivascular mononu- 
clear, lymphocytic, and plasma cell infiltrate with necrosis; 
ductal obstruction and edema; testicular interstitial edema; 
perivascular cerebral lymphocytic cuffing. 

Supportive; analgesics for pain; treat complications. 

A systemic infection caused by the mumps virus, an RNA 
paramyxovirus, mumps is transmitted by droplets and direct 
contact. Bilateral testicular involvement may lead to sterility; 
one of the most common causes of pancreatitis in children. 
A vaccine is available with measles and rubella (MMR). 



ID/CC A '20-year-old male college student presents with a productive 
cough, headache, malaise, runny nose, and fever. 

HPI He has a history of sore throat preceding the onset of the 
cough, which initially was nonproductive. 

PE VS: lever. PK: mild respiratory distress; auscultation reveals fine 
to medium rales over right lower lobe. 

Labs Gram stain of sputum negative; routine cultures of both blood 
and sputum negative. CBC: leukocyte count normal. Fourfold 
rise in complement fixation titer in paired sera; cold agglutinin 
titer > 1:128. 

Imaging CXR: patchy alveolar infiltrates involving right lower lobe; ap- 
pears worse than the clinical picture. 



MYCOPLASMA PNEUMONIA 



o 

c: 

s 

GO 



Gross Pathology Unilateral lower lobe pneumonia with lirm. red pulmonary 

parenchyma in affected areas. ^ 

m 

Micro Pathology Bronchial mucosa congested and edematous; inflammatory re- 
sponse consists of perivascular lymphocytes initially and PMNs 
later in infection. Organism lacks cell wall (thus penicillins and 

cephalosporins are ineffective). 

Treatment Erythromycin. 

Discussion Mycoplasma pneumoniae is the most common cause of primary 
atypical pneumonia. Transmission is by droplet spread; rapidly 
infects those living in close quarters. 






ID/CC A 50-year-old diabetic male presents with fever, pain, and a 
necrotizing swelling over his left leg. 

HPI His symptoms began about a week ago with redness and swelling 
of the left leg followed by bronze discoloration of the skin and 
the appearance of hemorrhagic bullae. 

PE Extensive cutaneous gangrene observed over left leg with many 
ruptured bullae; black necrotic eschar with surrounding ery- 
thema resembles a third-degree burn. 

Labs Swab staining reveals presence of chains of gram-positive cocci; 
culture isolated (J-hemolvtic group A streptococcus (Streptococcus 

pyogenes) . 



Micro Pathology 



Treatment 



Discussion 



Biopsy specimen reveals areas of necrosis in dermis and subcuta- 
neous fat. infiltration with PMNs, and vasculitis and thrombosis 
in vessels in the superficial fascia. 

Treatment includes rapid surgical excision of necrotic tissue in 

combination with appropriate antibiotics. 

Streptococcal gangrene is a group A streptococcal cellulitis that 
rapidly progresses to gangrene of the subcutaneous ussue and 
necrosis of the overlying skin; the disease process usually 
involves an extremity. Necrotizing fasciitis is also recognized as a 
polymicrobial infection that is caused by aerobes and anaerobes 
("SYNERGISTIC NECROTIZING CELLULITIS"). Infection spreads quickly 
through various fascial planes, the venous system, and 
lymphatics. Predisposing etiologies include surgery, trauma, 
and diabetes. 



NECROTIZING FASCIITIS 






ID/CC A 45-year-old white male undergoing chemotherapy for 

Hodgkin's lymphoma is brought to the emergency room by his 
wife because of shortness ol breaih and cyanosis. 

HPI For the past 3 months, he has been complaining of intermittent 
weakness, fever with chills, and foul-smelling, thick greenish 
sputum. 

PE VS: fever (38°C); tachypnea; tachycardia. PE: pallor; mild 

cyanosis; localized dullness with bronchial breathing; dimin- 
ished breaih sounds over left lower lobe. 



Labs 



Imaging 



Gross Pathology 



Micro Pathology 



CBC: leukocytosis with neutrophilia; anemia. Sputum culture 
reveals gram-positive, filamentous, partially acid-fast staining 
bacteria (due to Nocardia). 

CXR; nodular infiltrate in left lower lobe with air-fluid level 
(abscess) and left pleural effusion. 

Lung lesions or disseminated lesions (brain, liver, kidney, subcu- 
taneous tissue) consist of necrotic centers within regions of 
consolidation and abscess formation resembling pyogenic 
pneumonia. 

Consolidation of alveoli with pus formation (exudate of PMNs 
and fibrin) and surrounding granulomatous reaction. 



Treatment Six-month course of TMP-SMX; surgery. 

Discussion A chronic bacterial infection seen in diabetics, leukemia and 

lymphoma patients, and immunocompromised patients, nocar- 
diosis usually involves the lungs with possible dissemination to 
the brain, subcutaneous tissue, and other organs. It is caused by 
Nocardia asteroides, a branching, aerobic, gram-positive orga- 
nism that is weakly acid fast and is sometimes confused with 
Mycobacterium tuberculosis. 



NOCARDIOSIS 



<— > 

—i 

o 
cz 
oo 

o 

en 
m 
> 



ID/CC A 60-year-olcl male who was hospitalized following a stroke pre- 
sents with a high-grade fever with chills and obtundation. 

HPI He had been catheterized due to urinary incontinence and was 
receiving cephalosporin for treatment of aspiration pneumonitis. 

PE VS: fever. 

Labs Blood culture grew Enterococcus fecalis (morphologically indis- 
tinguishable from streptococci and immunologically similar to 
members of group D streptococci, the enterococci are metaboli- 
cally unique in their ability to resist heat, bile, and 6.5% NaCl); 
urine culture also isolated Streptococcus fecalis. 

Treatment Ampicillin with gentamicin (vancomycin can be substituted for 
ampicillin in patients with penicillin allergies). 

Discussion Enterococci constitute a relatively common cause of UTIs, 

wound infections, and peritonitis and intra-abdominal abscesses; 
they have also become an increasingly prominent cause of 
bacteremia, which usually originates from a focus in the urinary 
tract or abdomen. The incidence of nosocomial bacteremias 
caused by these organisms is also increasing, particularly in 
patients who have received cephalosporins or other broad- 
spectrum antibiotics. All clinically significant isolates should be 
subjected to testing for 3-lactamase production, high-level 
aminoglycoside resistance, and vancomycin resistance to deter- 
mine if an alternative therapy is necessary. Infections caused by 
enterococci that produce fi-lactamase are treated with an antimi- 
crobial agent that combines a penicillin with a P-lactamase 
inhibitor; infections caused by strains that are highly resistant 
to aminoglycosides are treated with vancomycin. 



NOSOCOMIAL ENTEROCOCCAL INFECTION 






"I 



ID/CC A 56-year-old white female is referred to an ophthalmologist for 
an evaluation of diminished visual acuity. 

HPI She has spent most of her adult life as a missionary in rural 
Senegal and Mali. She admits to chronic generalized itching, 
mostly while showering. 

PE Wrinkling and loss of elastic tissue in skin; marked hypopigmen- 
tation of shins; 2- to 3-cm, nonfixed, firm, nontender subcuta- 
neous nodules on iliac bones, knees, and elbows; chronic 
conjunctivitis, sclerosing keratitis, and chorioretinal lesions on 

eye exam. 

Labs CBC/PBS: eosinophilia. Fifty-milligram dose of diethylcarba- 

mazine produces severe pruritus, rash, fever, and conjunctivitis 
(positive Mazzotti reaction). 

Micro Pathology Skin biopsy at iliac crest shows microfilariae. 

Treatment Ivermectin; suramin. 

Discussion Onchocerciasis is caused by Onchocerca volvulus and is transmit- 
ted by the blackfly (SlMUUVM). which breeds near rivers; hence it 
is also known as "river blindness." 1-arvae migrate through sub- 
cutaneous tissue, producing painless soft tissue edema (CALABAR 
EDEMA); with time, subcutaneous nodules form and filariae ob- 
struct dermal h/mphatics, producing atrophy and hypopigmen- 
tation. Microfilariae concentrate in the eyes, leading to 
chorioretinitis and blindness. 



ONCHOCERCIASIS 










ID/CC A 9-year-old male is admitted for an evaluation of a suspected 
underlying immune deficiency. 

HPI He has been hospitalized and treated several times for recurrent 
life-threatening septicemia due to Streptococcus pneumoniae, 
meningococcus, and Haemophilus influenzae. Careful history 
reveals that a few years ago he underwent an emergency 
splenectomy following traumatic splenic rupture in a motor 
vehicle accident. 

PE Left paramedian postsurgical scar seen on abdomen. 

Labs Reduced IgM levels; reduced antibody production when chal- 
lenged with particulate antigens; PBS reveals Howell-Jolly 
bodies. 

Imaging US, abdomen: spleen is absent. 

Treatment Pneumococcal vaccine and prophylactic antibiotics (penicillin, 
amoxicillin, TMP-SMX). 

Discussion Patients who have undergone splenectomy or who are function- 
ally asplenic are at increased risk for overwhelming bacteremia; 
pathogens include organisms that possess a polysaccharide 
capsule, such as meningococcus, Staphylococcus, the DF2 bacillus, 
and, especially. Streptococcus pneumoniae and Haemophilus 
influenzae type B. Such functionally asplenic patients include 
individuals with sickle cell disease and those who have 
undergone splenic irradiation. Pneumococcal vaccine is 
indicated in all patients who have undergone splenectomy, 
particularly children and adolescents. 



OVERWHELMING POSTSPLENECTOMY INFECTIONS 






ID/CC A 12-year-old girl arrives in the emergency room with pain, 

swelling, and limited motion of her left hand; she also complains 
of fever and chills. 

HPI The girl was bitten by a cat yesterday while playing at a friend's 
house. 

PE Hand is ei yilicmalous, shiny, and markedly edematous; on 
palpation, hand is tender with fluctuation (cellulitis); limited 
passive and active motion; yellowish-green purulent fluid drains 
from wound; left epitrochlear and axillary lymphadenitis without 
lymphangitis. 

Labs Gram-negative rods with bipolar staining of abscess aspirate; 
catalase and oxidase positive (PasteurelUi multoada) . 

Imaging XR. plain: soft tissue swelling; no periostitis or erosions (vs. 
osteomyelitis). 

Treatment Incision and drainage, amoxicillin/clavulanate; tetracycline; 

penicillin. 

Discussion Pasteurelia multocida is the most common bacterium isolated 

from cat bite wounds and may progress to osteomyelitis. Human 
bite infections are most commonly caused by EikeneBa mrrodens 
and are treated with penicillin. 



o 

cz 

CO 

o 
i — i 

CO 

m 
3> 

t/1 



PASTEURELLA MULTOCIDA 



ID/CC A 44-year-old male archaeologist presents with high fever, 

malaise, intense headache, severe myalgia, and painful swelling 
in the inguinal region. 

HPI He recently returned from a trip to Arizona. 

PE VS: tachycardia. PE: drowsy looking; no meningeal signs; pustule 
seen at site of an insect bite on left upper arm: inguinal lymph 
nodes enlarged, fluctuant, and tender (buboes); no lesion on 
external genitalia. 

Labs CBC/PBS: normal; no malarial parasites. Gram-negative bacilli 
with "safety pin" appearance seen in aspirates from buboes; 
culture of aspirate reveals Yersinia pestis. 



Gross Pathology 



Enlarged lymph nodes are necrotic and suppurative; pneu- 
monic form shows lobar consolidation. 



Micro Pathology Numerous organisms in suppurative and necrotic lymph tissue. 

Treatment Streptomycin; gentamicin; doxycycline prophylaxis for close 
contacts; tetracycline. 



Discussion 



Plague is usually acquired alter contact with rodents and fleas in 
endemic areas (southwestern United States). Septic shock, 
pneumonia, DIG, and vascular collapse are life-threatening 
sequelae. Death rapidly ensues in the absence of treatment. 



PLAGUE 






ID/CC An 1 1-year-old white male presents with a high-grade fever, a 
productive, blood-tinged cough, mucoid sputum, and pleuritic 
left-sided chest pain of a few days' duration. 

HPI The child had previously been well and is fully immunized. 

PE VS: fever; tachypnea. PE: use of accessory respiratory muscles; 
central trachea; decreased left respiratory excursion; increased 
vocal fremitus in left infrascapular area with dullness to 
percussion; bronchia] breathing with coarse crackles heard over 
left lung area. 

Labs CBC: increased WBC count; preponderance of neutrophils. 
ABGs: hypoxemia without hypercapnia. Gram-positive 
diplococci in sputum; ct-hemolytic colonies of gram-positive 
diplococci (Streptococcus pneumoniae) on blood agar culture. 



Imaging 



Gross Pathology 



CXR: homogenous opacification of left lower lobe (LOBAR 
CONSOLIDATION) with small left pleural effusion. 

Consolidation of lung parenchyma passes through four stages: 
congestion and edema, red hepatization, gray hepatization, and 
resolution. 



Micro Pathology Vascular dilatation with hyperemia and alveolar edema; PMNs 
rich in purulent exudate; fibrin deposition; hardening of lung 
parenchyma with fibrin clotting inside alveoli (consolidation). 

Treatment Parenteral therapy with penicillin; monitor with radiologic 
imaging; supplemental oxygen for respiratory distress. 

Discussion S. pneumoniae is the most common cause of community- 
acquired pneumonia and produces typical lobar pneumonia. 



Atlas Links 



CD M-M2-035A, M-M2-035B, M-M2-035C, PG-M2-035 



PNEUMOCOCCAL PNEUMONIA 






i=! 
o 

cz 

on 

O 

i — i 

rn 

3> 

go 



ID/CC A 32-year-old HIV-positive male presents with progressively 
increasing dyspnea over the past 3 weeks. 

HPI He also complains of a dry, painful cough, marked fatigue, and 
a continuous low-grade fever. He has been noncompliant with 
cotrimoxazole prophylaxis. 

PE VS: fever; marked tachypnea. PE: pallor; generalized lym- 

phadenopathy; respiratory distress; intercostal retraction: mild 
central cyanosis; nasal flaring; coarse, crepitant rales auscultated 
at both lung bases. 



Labs 



Imaging 

Gross Pathology 
Micro Pathology 



ABGs: hypoxemia out of proportion to clinical findings. 

Pneumocystis am nit on methenamine silver stain of induced spu- 
tum or bronchoalveolar lavage; ELISA/Western blot positive for 
HIV. CBC: leukopenia with depressed CD4+ cell count. Serum 
LDH typically elevated. 

CXR: diffuse, bilaterally symmetrical interstitial and alveolar in- 
filtration pattern, predominantly perihilar; no lymphadenopathy 
or effusion. 

Congestion and consolidation of lungs with hypoaeration. 

Eosinophilic exudate in alveoli with multiple 4- to 6-mm cysts 
containing oval bodies (mf.rozoites) on lung biopsy or 
bronchial lavage; Pneumocystis abundant on Gomori 
methenamine silver stain. 



Treatment TMP-SMX; pentamidine; steroids for severe disease. 

Discussion Pneumocystis carinii pneumonia is an opportunistic infection that 
causes interstitial pneumonia in many immunocompromised 
patients. Traditionally it has been classified as a protozoan; 
however, P. carinii ribosomal RNA indicates that the organism is 
fungal. It is seen in the upper lobes in patients receiving inhaled 
pentamidine prophylaxis. Treat HIV patients prophylactically 
with TMP-SMX for P. carinii pneumonia if the CD4 count 
is < 200. 

Atlas Links EUSIIE M-M2-036A, M-M2-036B, M-M2-036C 



PNEUMOCYSTIS CARINII PNEUMONIA 



ID/CC A 25-year-old HIV-negative homosexual male presents with rectal 
burning, itching in the anal region, diarrhea, tenesmus, and a 
bloody, mucopurulent discharge per rectum. 

HPI One month ago he was hospitalized with severe febrile 

proctocolitis that was diagnosed as lymphogranuloma venereum. 
He has also been treated several times in the past for amebiasis 
and shigella colitis and admits to having receptive anal 
intercourse. Further history reveals that his most recent sexual 
partner has been suffering from urethral pain and discharge. 

PE Condylomata acuminata noted in perianal distribution; remain- 
der of physical exam normal. 

Labs Gram stain and culture of rectal swab reveals gram-negative 
diplococci identified as Neisseria gonorrhoeae on Thayer- 
Martin medium; urethral swab from partner also isolates 
N. gonorrhoeae. 

Imaging Sigmoidoscopy: proctitis with bloody mucopurulent discharge 
noted. 

Treatment Ceftriaxone and doxycycline (to treat likely concomitant 

chlamydial infection) for bodi patient and partner. Most apparent 
failures of correct antibiotic therapy are in fact due to reinfection; 
in resistant cases, spectinomycin, fluoroquinolones, or other 
cephalosporins can be used. 

Discussion The term "gay bowel syndrome" is used in reference to enteric 
and perirectal infections that are commonly encountered in 
immune-competent homosexual men; in homosexuals with HIV, 
opportunistic organisms play a more important role. Common 
etiologic agents include Chlamydia trachomatis, lymphogranu- 
loma venereum serovars, Neisseria gonorrhoeae, HSV, Treponema 
pallidum, human papillomavirus, Campylobacter species, Shigella, 
Entamoeba histolytica, and Giardia. 




n 



PROCTOCOLITIS 



ID/CC A 35-year-old male presents with high fever, malaise, headache, 
and a hacking cough productive of a small amount of mucoid 
sputum. 

HPI Me has two pet parrots at home who have recentiy shown signs 
of illness. 

PE VS: fever; bradycardia. PE: auscultation of chest reveals crepitant 
rales over both lower lung fields; splenomegaly with mild 
hepatomegaly noted; multiple purpuric macules seen over 
abdomen ("HORDER'S SPOTS"). 

Labs Greater than fourfold rise in complement-fixing antibody titer 
to a group antigen suggestive of infection with Chlamydia 
psittaci; definitive diagnosis of psittacosis was made from spu- 
tum by isolation of C. psittaci in pretreated tissue culture cells. 

Imaging CXR, PA: interstitial patchy, bilateral infiltrates. 

Gross Pathology Principal lesions found in lungs, liver and spleen. 



Micro Pathology 



Pulmonary lesion is an interstitial pneumonitis; mononuclear 
cells with ballooned cytoplasm containing inclusion bodies are 
observed. In the liver, focal necrosis of hepatocyte occurs along 
with Kupffer cell hyperplasia. 



Treatment Azidiromycin or clarithromycin. 

Discussion Psittacosis is an acute infection caused by Chlamydia psittaci: it 
is characterized primarily by pneumonitis and systemic manifes- 
tations and is transmitted to humans by a variety of avian 
species, principally psittacine birds (parrots, parakeets). A his- 
tory of contact with birds, particularly sick birds, or of employ- 
ment in a pet shop or in the poultry industry provides a clue to 
the diagnosis of psittacosis in a patient with pneumonia, espe- 
cially if bradycardia and splenomegaly are also present. 



PSITTACOSIS 



ID/CC A 54-year-old female being treated in the ER is noted to have 
developed progressively worsening abdominal pain and high- 
grade fever with chills. 

HPI She presented to the ER a few hours ago with colicky abdominal 
pain and was diagnosed with choledocholithiasis. 

PE VS: fever (39.5C), hypotension (BP 80/60); tachycardia 

(HR 120). PE: toxic-looking; icteric: abdominal exam reveals 
extremely tender RUQ with hepatomegaly. 

Labs CBC; leukocytosis with neutrophilia. LFTs: markedly elevated 
bilirubin, AST, ALT, alkaline phosphatase and GGT. Blood 
cultures grew Escherichia coli. 

Imaging CT, abdomen: multiple hepatic abscesses; distended gallbladder 
with perihepatic and perieliolecvstic fluid collections. 

Treatment Prolonged TV antibiotic therapy; emergent endoscopic (ERCP) 
or surgical biliary decompression; surgical drainage of the 
abscesses if no response to IV antibiotics. 

Discussion A pyogenic liver abscess is a pus-filled cavity within the liver 
caused by a bacterial infection, typically polymicrobial. The 
causes of liver abscess include abdominal infection such as ap- 
pendicitis, diverticulitis, or perforated bowel: sepsis; biliary tract 
infection; or liver trauma leading to secondary infection. The 
most common bacteria involved are E. coli, Klebsiella spp., 
Enterococcus, Staphylococcus spp., Streptococcus spp., and 
Bacteroides. Positive blood cultures are found in about half 
of patients with a pyogenic liver abscess and sepsis is a life- 
threatening complication. There is significant mortality even in 
treated patients and mortality is higher in those with multiple 
abscesses. 



o 

— i 
i — i 
o 
cz 

CO 

o 

I — I 

in 
m 



PYOGENIC LIVER ABSCESS 






ID/CC A 30-year-old dairy farm worker presents widi complaints of 
fever, headache, cough, pleuritic chest pain, and malaise. 

HPI His work at the dairy involves milking cows and looking after 
parturient cattle. 

PE VS: fever; tachypnea. PE: mild icterus; bilateral crackles on chest 
auscultation. 

Labs CliC: normal WBC count. Mild elevation of serum bilirubin and 
liver enzymes; greater than fourfold increase in complement- 
fixing antibody (against Coxiella burnetii) titer between acute 
and convalescent sera (IFA technique for early detection of spe- 
cific IgM Ab is die serodiagnostic method of choice); negative 
Weil-Felix reaction; C. burnetii isolated from sputum by inoculation 
of cultured human fetal diploid fibroblasts. 

Imaging CXR: right upper lobe rounded opacity that increased in size 
over a few days and cleared completely with treatment. 

Treatment Doxycycline is the first-line agent of therapy (erythromycin can 
also be used). 

Discussion Q fever is caused by the rickettsia-like organism Coxiella 

burnetii and produces the clinical picture of primary atypical 
pneumonia. Q fever differs from the odier human rickettsioses 
in that rash is absent and transmission is usually by the airborne 
route. C. burnetii localizes in die mammary glands and uterus of 
pregnant cattle, sheep, and goats, in which infection is mild or 
inapparent; infected placentas, postpartum discharges, and the 
feces of these animals are the principal sources of contaminated 
material in die environment. Humans acquire Q fever by 
inhaling aerosolized particles from such substances; particularly 
at risk are dairy and slaughterhouse workers. 



Q FEVER 



r 



ID/CC A 12-year-old while female is rushed to the emergency room 
because of numbness of the right foot and leg followed by 
fever and convulsions. The child refuses to drink any fluids 

(HYDROPHOBIA). 

HPI She had been camping 5 weeks ago. When questioned, her 

molher recalls that one night the child had apparendy stepped 
on a bat that bit her in the right foot. 

PE VS: no fever. PE: child is disoriented, hyperventilating, 

extremely agitated, and actively moving all four limbs; thus 
difficult to restrain; no meningeal signs; fundus normal; 
saliva viscous and foaming. 

Labs LP: lymphocytic pleocytosis with mildly elevated proteins 

and normal sugar in CSF. Positive rabies antigen in corneal 
scrapings. 



Micro Pathology 



Treatment 



Discussion 



Characteristic cytoplasmic inclusion bodies (Negri bodies) in 
corneal scrapings or Amnion's horn. 

Supportive; almost always fatal; prevent with vaccine; postexpo- 
sure prophylaxis with diploid cell vaccine and human rabies 
immune globulin (HR1G). 

Rabies is a fatal viral disease that is transmitted to humans by the 
biles of bats, raccoons, skunks, foxes, coyotes, dogs, and cats. 
Rabies virus is an enveloped, single-stranded RNA virus. Rabies 
has a long incubation period (approximately 3 to 8 weeks); 
death usually results from respiratory failure. 



a 

o 



o 
K 

m 



Atlas Link tnc^iii M-M2-041 



RABIES 









ID/CC A 27-year-old male researcher presents with sudden-onset fever, 
chills, headache, a skin rash, and painful swelling of multiple 
limb joints. 

HPI Careful history reveals that he was bitten by a rat in his labora- 
tory a few days ago; the bite wound has now healed. 

PE VS: fever. PE: morbilliform rash noted over extremities, particu- 
larly the hands and feet; painful swelling and restriction of 
movement noted over both wrist and knee joints. 

Labs CBC: leukocytosis. Streptobacillus moniliformis isolated from 
blood and synovial fluid of inflamed joints; agglutinins to S. 
moniliformis demonstrated in significant titers. 

Treatment Amoxicillin/clavulonic acid (doxycycline can also be used). 

Discussion Rat bite fever, which is caused by Streptobacillus moniliformis, 
is an acute febrile illness that is usually accompanied by a skin 
rash; most cases result from the bites of wild or lab rats, although 
mice, squirrels, weasels, dogs, and cats may also transmit the 
disease by bites or scratches. The disease is called Haverhill 
fever when S. moniliformis is transmitted by drinking rat- 
excrement-contaminated milk. Distribution is probably 
worldwide, with most cases occurring in crowded cities 
characterized by poor sanitation. 



RAT BITE FEVER 



ID/CC A 30-year-old male who lives in the western part of the United 
States presents with high fever, shaking chills, severe headache, 
myalgias, and diarrhea. 

HPI He reports having had similar symptoms 10 days ago that lasted 
for 4 to 5 days, followed by defervescence accompanied by 
drenching sweats and marked prostration. He had been hiking 
in a tick-infested forest until about a week before the develop- 
ment of symptoms. 

PE VS: fever. 

Labs Spirochetes found on thick smears of peripheral blood obtained 
during febrile period and stained with Wright or Giemsa stain. 

Treatment Doxycycline is the drug of choice (erythromycin may also be 
used). 

Discussion Relapsing fever is an acute louse-borne or tick-borne infection 
that is caused by blood spirochetes of the genus Borrelitr, it is 
characterized by recurrent febrile episodes separated by asymp- 
tomatic intervals. Unlike other spirochetes, the etiologic agent 
can readily be detected with Giemsa slain or Wright's stain. 
B. recurrentis is the cause of louse-borne relapsing fever. 
whereas a variety of different species produce the tick-borne 
disease. In the United States, the predominant species are 
B. kermsii and B. luricatae. Most patients experience the 
Jaa im h-Herxheimer reaction within the first 2 hours of treatment 



RELAPSING FEVER 



1 — 1 




z 




-n 




1 1 1 




(~i 




13 




O 




CT 




L/> 




o 




i — i 




U1 





> 

E/i 






ID/CC A 6-year-old male presents with fever, intense headache, myalgia, 
dry cough, and a rash that began peripherally (on his wrists and 
ankles) but now involves the entire body, including the palms 
and soles. 

HPI The child lives in North Carolina and indicates that he was 

bitten by an insect a few weeks ago while playing in the woods 
near his home. 

PE VS: fever. PE: lethargy; ill appearance: petechial rash all over 

body, including palms and soles. 



Labs 



Gross Pathology 



Micro Pathology 



( KC: thrombocytopenia: prolonged bleeding and clotting time. 
Positive Hess capillary test (RiMi'i.t.-I.i l DK i-hknomknON); 
positive Proteus OX 19 and OX2 Weil-Felix reaction: specific 
antibodies to Rickettsia rickettsii with positive complement 
fixation. UA: proteinuria; hematuria. 

Hemorrhagic necrosis in brain and kidneys; nodular formation 
in glia. 

Inflammatory lymphocytic and plasma cell perivascular infiltra- 
tion; endothelial edema with abundant rickettsiae; microthrom- 
bus formation with necrotic vasculitis. 



Treatment Doxycycline or chloramphenicol. 

Discussion Rickettsia rickettsii is the causative organism of Rocky Mountain 
spotted fever; Dermacentor, a wood tick, is the vector. The orga- 
nism's tropism for endothelial cells results in vasculitis, edema, 
thrombosis, and ischemia. Ironically, Rocky Mountain spotted 
fever is endemic to the East Coast of the United States. 



ROCKY MOUNTAIN SPOTTED FEVER 



ID/CC A 5-month-old male infant is brought to the pediatric clinic with 
wheezing and respiratory difficulty of 3 hours' duration. 

HPI He has had rhinorrhea, fever, and cough and had been sneezing 
for 2 days prior to his visit to the clinic. 

PE VS: tachypnea. PE: nasal flaring; mild central cyanosis; accessory 
muscle use during respiration; hyperexpansion of chest; expira- 
tory and inspiratory wheezes; rhonchi over both lung fields. 

Labs CBC/PBS: relative lymphocytosis. ABGs: hypoxemia with mild 
hypercapnia. Normal flora on bacterial culture of sputum; 
respiratory syncytial virus (RSV) demonstrated on viral culture 
of throat swab. 

Imaging CXR; hyperinflation; segmental atelectasis; interstitial infiltrates. 

Treatment Humidified oxygen; bronchodilators; aerosolized ribavirin. 

Discussion RSV is the most common cause of bronchiolitis in infants under 
2 years of age; other viral causes include parainfluenza, in- 
lluenza. and adenovirus. Infections typically occur during the 
fall and winter months. Transmission occurs via close contact 
with contaminated fomites but can also occur after coughing or 
sneezing. The majority of infections occur during an RSV 
epidemic. 



O 

cz 
on 

o 

h— I 
LO 

m 
3> 
IS) 



RSV PNEUMONIA 



ID/CC A 4-month-old girl brought in for a well-child visit is found to be 
low in weight and height for her age and to have lens opacities 
(due to congenital cataracts). 

HPI Her mother had a skin rash and fever during her first trimester. 
The mother states that when the child was born, she too had a 
rash like a "blueberry muffin" and was jaundiced. 

PE Deaf and globally retarded; malnourished; microcephaly and 
bulging anterior fontanelle; microphthalmia with unilateral left 
cataract: discrete black, patchy pigmentation found in retina on 
funduscopic exam; hepatosplenomegaly; machinery murmur 
heard at second intercostal space on left sternal border (due to 
patent ductus arteriosus) . 

Labs CBC/PBS: leukopenia; thrombocytopenia. Increased serum 

bilirubin (both direct and indirect); rubella virus isolated from 
urine and saliva; markedly increased IgM specific antibody for 
rubella. 

Imaging XR, plain: radiolucent (lytic) bone lesions (metaphyseal). 

Treatment None. 

Discussion Congenital rubella, transmitted in utero, is caused by rubella 

virus, a single-stranded KNA togavirus. In children and adults it 
is a transitory and unremarkable disease. If acquired in utero it 
has devastating consequences. 



RUBELLA— CONGENITAL 



ID/CC A 10-year-old female Asian immigrant presents with a low-grade 
fever and coryza of 3 days' duration. 

HPI She also complains of arthralgias and a skin rash that began on 
her face and spread to her trunk. Her mother says she cannot 
remember any details of her vaccination history. 

PE VS: fever. PE: maculopapular rash over face and trunk; enlarged 
postauricular, posterior cervical, and occipital lymph nodes. 

Labs CBC: leukopenia; thrombocytopenia. Rubella virus hemaggluti- 
nation inhibition test demonstrates fourfold rise in titer to 1:32. 



Gross Pathology 



Erythematous skin rash resembling rubeola measles but lighter 
in color and more discrete; similar distribution pattern in both. 



Treatment Symptomatic treatment. 

Discussion Rubella (German measles) is caused by a togavirus. Live attenu- 
ated rubella virus vaccine (part of MMR) should be given to all 
infants and to susceptible girls before menarche. The course of 
illness is self-limiting and mild; in females the major implication 
is the potential for congenital rubella syndrome. Females with 
rubella can get polyarthritis secondary to immune complex 
deposition. 

Atlas Link □ I IM2-026 



RUBELLA (GERMAN MEASLES) 



<— > 

—i 
i—i 

o 

cz 
on 

o 

H- ! 
OO 

m 

> 
on 






ID/CC A 14-year-old male who is known to have sickle cell anemia pre- 
sents Willi throbbing pain, redness, and swelling ol the right 
thigh. 

HPI The patient also complains ol lever and chills of 1 week's dura- 
tion. He has a lew pet turtles at home. 

PE VS: fever; tachycardia. PE: pallor; redness, swelling, and tender- 
ness over right thigh; effusion denionsiiaied in right knee joint; 
limitation of range of motion of right knee. 

Labs CBC: leukocytosis; elevated ESR. PBS: irreversible sickling; 
blood culture reveals Salmonella typhimurium (most com- 
mon); organism also isolated from pus aspirated from right 
femur (diagnostic of osteomyelitis). 

Imaging Nuc: increased uptake in metaphyseal region of right femur. XR 
(usually normal during the first 10 days of illness) may reveal 
changes of bone resorption, detached necrotic cortical bone 
(SEQUESTRUM), and laminated periosteal new-bone formation 

(IMOU'CRUM). 

Gross Pathology Dense, pale, sclerotic-appearing area in shaft. 

Micro Pathology Changes include suppurative and ischemic destructive necrosis, 
fibrosis, and ultimate bone repair. 

Treatment Parenteral antibiotics, with fluoroquinolones being first-line 
agents (third-generation cephalosporins may also be used). 

Discussion A striking association has been noted between diseases produc- 
ing hemolysis (e.g., sickle cell anemia, malaria, and bartonel- 
losis) and salmonella infections; elderly patients with impaired 
host defense mechanisms, those with hcpatosplenic schistosomi- 
asis, and AIDS patients are also at increased risk of severe and 
recurrent salmonella bacteremia. Salmonella osteomyelitis in 
sickle cell patients presents primarily in young individuals and 
typically affects long bones. It is believed that die functional 
asplenic state found in most sickle cell patients contributes 
to the increased prevalence of salmonella osteomyelitis. 



SALMONELLA SEPTICEMIA WITH OSTEOMYELITIS 



ID/CC Ail Asian refugee family comprising a 30-year-old man, his wife, 
and i wo schoolchildren present with complaints of severe 
itching over their entire bodies except for their face; the itching 
increases during the night. 

HPI The male family members also report penile and scrotal skin 
lesions. The family is of low socioeconomic status and lives in a 
single room under crowded conditions. 

PE Papulovesicular lesions; "burrows" seen in the dorsal interdigi- 
lal web spaces and flexor aspects of both wrists; lesions also seen 
around elbows, anterior axillary folds, periumbilical area, lower 
buttocks, and thighs; face was spared; scrotal and penile lesions 
seen in male members were nodular and reddish. 

Labs Female adult mite was seen with a hand lens when teased out of 
her burrow with a needle. 

Treatment Apply lindane or permethrin (lindane is contraindicated in 
small children and in pregnant women). All family members 
must be treated; clothing, linen, and the like should be boiled 

and washed: fingernails should be trimmed. Use antihistamines 
or calamine lotion to help control itching. 

Discussion Scabies is caused by infestation with Sarcoptes scabiei, a mite 

that bores into the corneal layer of the skin, forming burrows in 
which it deposits its eggs. The scabies organism does not survive 
for more than 48 hours away from the host; modes of transmis- 
sion include close contact with infected individuals, unsanitary 
conditions, and sexual contact. In adults, certain areas of the 
body are generally spared, including the face, scalp, and neck. 



Atlas Links BED I M-M2-049 



mcMzl MC-186 



SCABIES 



o 




ID/CC A 10-year-old white female complains of difficulty swallowing, 
pain in both ears, and fever of 1 week's duration; she also 
complains of an extensive skin rash. 

HPI The child is fully immunized and has been well until now. 

PE VS: fever. PE: extensive erythematous rash ( "goose-pimple 
sunburn") on neck, groin, and axillae; desquamation and 
peeling of fingertips; circumoral pallor; lines of hyperpigmenta- 
tion with tiny petechiae (Pastia's sign) in antecubital fossae; 
bright red lingual papillae superimposed on white coat 
( "strawberry TONGUE") ; pharyngitis with exudative tonsillitis; 
cervical lymphadenopalhy; normal eardrums. 

Labs CBC: leukocytosis with neutrophilia. Group A B-hemolytic 

Streptococcus pyogenes on throat swab and culture; elevated ASO 
titer. 

Gross Pathology Toxin-induced vasodilation; complications include otitis media, 
pneumonia, glomerulonephritis, osteomyelitis, and rheumatic 
fever. 

Micro Pathology Inflammatory polymorphonuclear epidermal infiltrate; intersti- 
tial nephritis; lymph node hyperplasia. 

Treatment Penicillin; erythromycin. 

Discussion Scarlet fever is a streptococcal infection that is characterized by 
morbilliform rash due to hypersensitivity to erythrogenic toxin. 

Atlas Links L'Jcmzi MC-187A, MC-187B 



SCARLET FEVER 



ID/CC A 27-year-old Peace Corps volunteer working in the Congo is 
sent home after developing fever, sweats, and abdominal pain 

that have not responded to antimalarial treatment. 

HPI Five weeks ago, he developed severe itching and a macular rash 

(swimmer's itch) after swimming in a nearby pond. 

PE VS: fever. PE: moderate enlargement of liver and spleen; lender 
abdomen but no peritoneal irritation. 

Labs CBC/PBS: marked eosinophilia. Characteristic large parasite 
eggs with lateral spines may be found in stool specimen. 

Imaging Sigmoidoscopy: swollen and erythematous mucosa; many small 
ulcerations. CT/US, abdomen: hepalosplenomegaly; portal vein 
dilatation. 



Gross Pathology 



Skin and liver sites of principal lesions in acute stage; eggs may 
be found in liver, lungs, intestines, pancreas, spleen, urogenital 
organs, and brain; chronic stage characterized by granuloma 
formation in bladder and liver (periportal fibrosis). 



Micro Pathology Granulomatous reaction and fibrosis. 

Treatment Praziquantel. 

Discussion Schistosomiasis is among the most common parasitic diseases in 
ihe world: infection with Schistosoma mansoni or S.japonicum is 
acquired by swimming in snail-infested ponds and lakes. Long- 
standing infection may lead to noncirrhotic portal fibrosis and 
portal hypertension. Also known as bilharziasis. 

Atlas Link LUcSaTJ M-M2-051 



o 



SCHISTOSOMIASIS 






ID/CC A 12-year-old immigrant from the Middle East presents with 
terminal hematuria, dysuria, and increased frequency of 
micturition. 

HPI He remembers having played and bathed in snail-infested 

streams while he was in his native country; on one occasion he 
had developed an intensely pruritic skin eruption after bathing 
in one such stream ("cercarial dermatitis"). 

PE Pallor noted. 

Labs UA: hematuria; mild proteinuria and sterile (abacterial) pyuria. 
Microscopic exam of urine and rectal biopsy reveals presence of 
ellipsoid eggs with a sharp terminal spine containing a miracid- 
ium surrounded by a thick, rigid shell. 

Imaging XR: bladder wall calcification. 

Treatment Praziquantel, metrifonate. 

Discussion Three major species exist. Schistosoma mansoni, S. japonicum, and 
S. haematobium infect humans. .S". mansoni is found in Africa, the 
Arabian Peninsula, South America, and parts of the Caribbean; 
S. japonicum is found in Japan, China, and the Philippines; and 
S. haematobium is found in Africa and the Middle East. 
Transmission of schistosomiasis cannot occur in the United 
States because of the absence of the specific freshwater snail 
that is an intermediary host. In .S'. haematobium infection, the 
principal symptoms are terminal hematuria, dysuria, and fre- 
quent urination; hydronephrosis, pyelonephritis, and squamous 
cell carcinoma of the urinary bladder may develop as 
complications. In S. mansoni and S. japonicum infection, 
manifestations may include fever, malaise, abdominal pain, 
diarrhea, or hepatosplenomegaly. Presinusoidal hepatic trapping 
of eggs and the consequent granulomatous reaction induce 
portal hypertension. 



SCHISTOSOMIASIS— URINARY 






ID/CC A 36-year-old male executive comes to the emergency room 
because of the development of sudden nausea, vomiting, and 
diarrhea with blood and mucus (dysentery) as well as crampy 
abdominal pain for 2 clays. 

HPI He had just returned from a business trip in South America. 

PE VS: low-grade fever. PE: mild dehydration; hyperactive bowel 
sounds; tender abdomen without definite peritoneal irritation. 

Labs Leukocytes on stool examination; Shigella isolated on stool cul- 
ture; on microbiology, organism does not ferment lactose and is 
not motile. 

Micro Pathology Shigella enterotoxin acts by activating adenylate cyclase; 
organism invades intestinal mucosa. 

Treatment Rehydration with antibiotic therapy (ampicillin orTMP-SMX). 

Discussion Shigellosis outbreaks occur primarily in areas with overcrowding 

and poor hygiene (fecal-oral transmission): arthritis, conjunctivitis, 
and urethritis (Reiter's SVNDRf >mi- ) may be complications in 
HLA-B27-positive individuals. Like Salmonella, Shigella causes 
bloody diarrhea by invading the intestinal mucosa, causing 
intestinal ulceration and inflammation. 



3 

o 

cr 
oo 

a 

i— i 

m 
> 

m 






SHIGELLOSIS 



ID/CC A 56-year-old hospitalized male is found to have an abrupt-onset 
high-grade fever with chills a few hours after he underwent 
nephrolithotomy. 

HPI He was diagnosed with chronic nephrolithiasis with recurrent 
UTIs: a surgery intern also noted poor urine output. 

PE VS: fever; tachycardia: hypotension; tachypnea. PE: confused 

and disoriented; hyperventilating: diaphoresis; hands warm and 
pink with rapid capillary refill; pulse bounding: on chest 
auscultation, air entry found to be bilaterally reduced. 

Labs CBC: leukocytosis with left shift; neutrophils contain toxic 
granulations, Dohle bodies, and cytoplasmic vacuoles; band 
forms > 10%; thrombocytopenia. Prolongation of thrombin 
lime, decreased fibrinogen, and presence ol D-dimers (suggest- 
ing DIG); raised BUN and creatinine. ABGs: metabolic acidosis 
(increased anion gap due to lactic acidosis) and hypoxemia 
(due to ARDS). Blood and urine culture yields Escherichia coli. 

Imaging CXR: evidence of noncardiogenic pulmonary edema (ARDS). 

Treatment IV antibiotics (with adequate gram-negative coverage): manage- 
ment of multiorgan failure (azotemia. ARDS, and DIG). 

Discussion Almost anv bacterium can cause a bacteremia, including E. coli 
(most common), Klebsiella, Proteus, Pseudoinonas (associated with 
antibiotic therapy and burn wounds), Bacteroides fragilis (causes 
of anaerobic septicemias). Staphylococcus aureus. Streptococcus 
pneumoniae, and pediatric septicemia due to /;. coli and 
Streptococcus agalacliae. Gram-negative bacteria release endotox- 
ins; the release of endotoxin into the circulation leads to the 
activation of macrophages and monocytes, which in turn release 
cyiokines. These cytokines trigger cascade reactions that lead 
to the clinical and biochemical manifestations of the sepsis 
syndrome. 

Atlas Links dEEE M-M2-054A, M-M2-054B 



SHOCK— SEPTIC 



ID/CC A 37-year-old gardener complains of lumps with red streaks on 
his arm and swelling of the axillary lymph nodes. 

HPI Two weeks ago, he pricked his hand with a thorn while pi lining 
roses. A nodule then formed which subsequendy ulcerated and 
filled wilh pus. 

PE Nonpainful nodular lesion on dorsum of hand with ulcer forma- 
tion and suppuration; tender, palpable inflammation and hard- 
ening of lymph vessels (1 vuill WGITIS); swelling, inflammation. 
and suppuration of lymph nodes (LYMPHADENITIS); nonulceraied 
satellite nodules along course of lymphatics. 

Labs Cigar-shaped budding cells (Sporotiirix SCHENCKIl) visible in pus; 
diagnosis confirmed by culture of aspirate of nodule. 

Gross Pathology Nonpainful, soft, ulcerated nodule at inoculation site 

(SPOROTRK.IH n H t UANCRE): may extend to deep tissues and 
bone with osteitis and synovitis. 



Micro Pathology 



Usually area of suppuration with polymorphonuclear infiltrate 
surrounded by granulomatous reaction of varied intensity with 
epithelioid and giant cell formation; chlamydospore asteroid 

bodies present. 



Treatment Itraconazole; potassium iodide. 

Discussion Also called "rose gardener's disease," sporotrichosis is a fungal 
infection caused by Sporothrix schenckii, a dimorphic fungus 
that lives on vegetation. It is typically transmitted by ;i thorn prick 
and causes localized infection with few systemic manifestations. 

Atlas Link I i J M-M2-055 



o 
— { 
i— i 
o 
cz 
tn 

O 
i— i 
on 
rn 
> 
t/i 
m 



SPOROTRICHOSIS 



ID/CC A 7-year-old girl is seen by the embassy doctor in Nigeria for ab- 
dominal pain, diarrhea, fever, dry cough, and marked dyspnea 
of 2 weeks' duration. 

HPI She is the daughter of an American diplomat working in 

Nigeria. Despite her parent's admonitions, she frequently walks 
barefoot. 

PE VS: fever. PE: moderate respiratory distress; no cyanosis; no 

clubbing; coarse, crepitant rales and wheezing beard over both 
lung fields; mild abdominal tenderness. 

Labs CBC/PBS: marked eosinophilia. Typical motile rhabditiform 

larvae on sputum exam as well as in freshly passed stool; positive 
filarial complement fixation test. 

Imaging CXR: bilateral, transient migratory infiltrates. 

Gross Pathology Pneumonitis produced by migration of larvae through respiratory 
tract. 

Treatment Ivermectin, thiabendazole. 

Discussion Strongyloidiasis is seen in (he presence of poor hygiene and in 
tropica] countries. Larvae penetrate the skin, gaining entrance 
to the venous system and to the lungs, and then ascend to enter 
die Gl tract. 

Atlas Link f.Zl n M-M2-056 



STRONGYLOIDIASIS 



ID/CC A 54-year-old white female complains of spiking fever, chills, 
loss of appetite, several bouts of diarrhea, and right upper 
quadrant pain. 

HPI Ten days ago she underwent an apparently uncomplicated 

emergency surgery for suppurative cholecystitis and was subse- 
quently discharged and sent home. 

PE VS: fever. PE: pallor; slight icterus: pain on percussion of right 
costal region: well-healed surgical wound with DO evidence of 
infection: liver not palpable; crepitant rales on right lung base. 

Labs CBC: elevated WBC count (17,000) with predominance of 
neutrophils. 

Imaging CXR: elevated right hemidiaphragm; slight right pleural 

effusion. US/CT: complex fluid collection below diaphragm. 

Treatment Percutaneous drainage under ultrasonic or fluoroscopic 

guidance followed by regular blood and radiologic exams; 
surgical exploration and drainage. 

Discussion Subdiaphragmatic abscess most commonly occurs after abdomi- 
nal surgery, mainly with septic, emergency procedures; it typi- 
cally presents 1 week or more postoperatively. 



SUBDIAPHRAGMATIC ABSCESS 



ID/CC A 6-week-oId male, the son of a prostitute, is brought to the 
family doctor because of persistent, sometimes bloody 
mucopurulent nasal discharge, anal ulcers, and a generalized 
rash. 

HPI The child was delivered at home, and the mother did not 

receive any prenatal care. 

PE Weak-looking, icteric infant with hoarse cry; does noi move right 
limb (pseudoparalysis); bloody purulent discharge evident at 
nares; generalized lymphadenopathy; hcpatosplenomegaly; 
maculopapular rash with desquamation on back and buttocks; 
bullae on hands and feet. 

Labs CBC: anemia. VDRL in both mother and child positive; direct 

hyperbilirubinemia: negative Coombs' test: Treponema pallidum 
seen on nasal exudate and anal ulcers. 



Imaging 



Gross Pathology 



XR. plain: periostitis of long bones: bilateral moth-eaten lesions; 
focal defect in proximal tibial epiphysis with increased density of 
epiphyseal line (Wimberger's ska i. 

Pathologic features seen if neonatal disease is left untreated in- 
clude syphilitic chondritis and rhinitis (causes saddle-nose 
deformity), pathologic fractures, bowing of the tibia (SABER 
MUM. V-shaped incisors (HUTCHINSON'S TEETH), multicuspid 
molars (MULBERRY MOLARS), interstitial keratitis, and deafness. 



Treatment Penicillin. 

Discussion Treponema pallidum is a spirochete; in utero vertical transmission 
occurs from an infected mother to the fetus. Congenital syphilis 
occurs maximally during 16 to 36 weeks of gestation and may 
be the cause of stillbirth. Ii is preventable if the mother has 
received adequate treatment. 



SYPHILIS— CONGENITAL 



ID/CC An 18-year-old while male presents with a painless ulcer on his 
penis. 

HPI He admits to having had unprotected intercourse with a prosti- 
tute 3 weeks ago. 

PE Painless, single, rounded, firm papule with well-defined margins 
on dorsal aspect of glans penis that ulcerates ("hard chancrk"); 
nontender, rubbery bilateral inguinal lymphadenopathy. 

Labs Treponemes on dark-field examination of exudate from chan- 
cre; VDRL positive; FTA-ABS positive; ELISA for HIV negative. 

Gross Pathology A 1.2-cm ulcerated papule with rolled edges and induration; re- 
gional lymphadenopaihy. 

Micro Pathology Capillary dilatation with plasma cell, PMN, and macrophage in- 
filtration; fibroblastic reaction. 

Treatment Benzathine penicillin G 1M. 2.4 ML' single dose. 

Discussion An STD caused by Treponema pallidum, a spirochete, primary 

syphilis is characterized by the appearance of a painless chancre 
in the area of inoculation. If left untreated, secondary and terti- 
ary syphilis may ensue. Other STDs, such as AIDS, are more 
prevalent in patients with syphilis. 

Atlas Link wcrazi Z-M2-059 



o 

o 

cz 
1/1 

I — I 
t/1 
m 

> 






SYPHILIS— PRIMARY 



ID/CC A 23-year-old female presents wilh a nonpruritic skin eruption, 
hair loss, and generalized fatigue and weakness. 

HPI She admits to having had multiple sexual partners and 

unprotected sex. She has had two spontaneous abortions. 

PE Extensive raised, copper-colored, maculopapular, desquamative 
rash on palms and soles; generalized nontender lymphadenopathy 
with hepatosplenomegaly; large, pale, coalescent, flat-topped 
papules and plaques in groin (CONDYLOMATA lata); dull, erythe- 
matous mucous patches in mouth: hair loss (ALOPECIA) in tail of 
eyebrows. 

Labs Skin lesions, mucous patches in mouth, and condylomata lata 
positive for treponemes; positive VDRL; positive FTA-ABS; 
ELISA negadve for HIV; CSF VDRL negative. 

Treatment IM benzathine penicillin G. 

Discussion Sexual partners must be treated. 

Atlas Links i.uctgz\ IM2-027A, IM2-027B 



|3 SYPHILIS— SECONDARY 



ID/CC A 54-year-old man presents with ataxia, mental status changes, 
grossly deformed ankle joints, and shooting pains in his 

extremities. 

HPI He remembers having had a "boil" on his penis (primary 

syphii n u ( a i ANCRE) many years ago that went away by itself. He 
also recalls having had a scaling rash on the soles of his feet and 
die palms of his hands (due to secondary syphilis) some lime 
ago. 

PE Painless subcutaneous granulomatous nodules (GUMMAS); 
reduced joint position and vibration sense in both lower 
extremities (due to bilateral dorsal column destruction); loss of 
deep tendon reflexes in both lower limbs; loss of pain sensation 
and deformed ankle and knee joints with effusion (Charcot's 
NEUROPATHIC ARTHROPATHY): broad-based gait; positive Romberg's 
sign (due to sensory ataxia); pupillary light reflex lost but 
accommodation reflex retained (ARGYLL ROBERTSON PUPILS}. 

Labs Positive VDR1. and Treponema pallidum hemagglutination assay 
(TP-HA) . LP: pleocytosis and increased proteins in CSF; VDRL 
positive. Normal blood glucose levels. 

Imaging CXR: "tree-bark calcification" of ascending aorta. 

Gross Pathology Obliterative endarteritis and meningoencephalitis. 

Micro Pathology Proliferation of microglia; demyelini/.ation and axonal loss in 
dorsal roots and columns. 

Treatment Penicillin. 

Discussion Tabes dorsalis usually develops 15 to 20 years after initial 

infection. There may also be visceral involvement (can cause 
neurogenic bladder). 

Atlas Link n~Y~\ 1 IM2-028 



SYPHILIS— TERTIARY (TABES DORSALIS) 



ID/CC A 12-year-old white male presents with stiffness of the jaw and 
neck along with inability to swallow. 

HPI Twelve days ago lie stepped on a rusty nail, which produced a 
small puncture wound; the area is now red, hard, and swollen 
with pus. He has been experiencing tingling sensations and 
spasms in his calf muscles. He has not received any immunizations 
within the past 10 years. 

PE Jaw muscle rigidity ( trismus); facial muscle spasm (risus 
SARDonicus); dysphagia; neck rigidity; normal deep tendon 
reflexes; profuse sweating; patient alert, apprehensive, restless, 
and hyperactive during PE; loud noise elicits painful spasms of 
face, neck, abdomen, and back, the latter producing 
opisthotonos. 

Labs CBC, CSF, blood chemistries normal. 

Gross Pathology There may be fractures of ribs or vertebrae with sustained 
spasms. 

Treatment Surgical debridement of wound; tetanus immune globulin intra- 
muscularly or intrathecally; diazepam; phenobarbital; tetanus 
toxoid; penicillin IV. 

Discussion Tetanus is caused by tetanospasmin, a neurotoxin produced by 
Clostridium tetani, an obligate anaerobic, spore-forming, gram- 
positive rod; the toxin blocks the release of the inhibitory neu- 
rotransmitter glycine in the anterior horn cells. Tetanus often 
occurs in IV drug abusers; neonates of nonimmunized mothers 
may become infected through the umbilical cord stump. The 
disease may occur even years after injury or infection and may 
also involve the autonomic nervous system (arrhythmias, 
high/low blood pressure). 



Atlas Link 



W M-M2-062 



TETANUS 



ID/CC A 15-day-old infant is brought by his mother to the pediatric 

emergency room in a state of marked muscle rigidity and spasm. 

HPI The mother is illiterate and did not receive any prenatal care: 
the delivery was conducted at home and, according to her, was 
uneventful and full term. The child did not receive any 
immunizations; directed questioning reveals that he has been 
crying excessively for the past 2 weeks and has not been feeding 
normally. 

PE Extremely ill-looking infant in a state of generalized rigidity and 
opisthotonus; on slightest touch or noise, spasm worsens and he 
develops a stridor and becomes cyanosed. 

Labs Diagnosis is largely clinical; culture of umbilical stump yields 
Clostridium letani. 

Treatment Ventilatory assistance; supportive management; maintenance of 
nutritional, fluid, and electrolyte balance; tetanus antitoxin; 
control of tetanic spasms with diazepam. 

Discussion Tetanus neonatorum accounts for a considerable proportion of 
infant deaths in developing countries, primarily owing to the 
lack of availability of good prenatal care (no tetanus immuniza- 
tion); untrained birth attendants in rural areas use contaminated 
material to cut or anoint the umbilical cord. Tetanus is caused by 
Clostridium letani, a gram-positive, motile, nonencapsulated, 
anaerobic, spore-bearing bacillus; its effects are mediated 
through production of a powerful neurotoxin (tetanospasmin). 
The toxin acts principally on the spinal cord, altering normal 
control of the reflex arc by suppressing the inhibition regularly 
mediated by die internuncial neurons. 



— i 
>—< 
o 

o 

I— I 

CO 

m 

3> 



TETANUS NEONATORUM 



ID/CC A 40-year-old male who recently went hiking in a forest in the 
western United States presents with symmetric weakness of the 
lower extremities that has now progressed over the past few days 
to involve the trunk and die upper arms. 

HPI The patient does not report any sensory symptoms. 

PE Higher mental functions intact; symmetric flaccid paralysis with 
an ascending pattern of spread noted; no sensory loss demon- 
strated; <^n careful examination of hairy areas of the body, a tick 
was found embedded in the scalp. 

Labs LP: CSF normal. EMG: nerve conduction velocity and compound 
muscle action potentials decreased. 

Treatment Tick was detached without being squeezed, and this led to 
resolution of symptoms over the next few days. 

Discussion Feeding ticks may elaborate a neurotoxin that causes tick paraly- 
sis; symmetric weakness of the lower extremities progresses to an 
ascending flaccid paralysis over several hours to days, although 
the sensorium remains clear and sensory function is normal. 



TICK PARALYSIS 



ID/CC A 40-year-old male diagnosed with AIDS presents with a severe 
headache. 

HPI He suffered a grand mat seizure 2 hours before his arrival in the 
emergency room. He denies any history of seizures and adds 
that he has many pets, including cats. 

PE Generalized lymphadenopathy: bilateral papilledema; left-sided 
hemiparesis with lnperactive deep tendon reflexes on lefi side; 
positive Babinski's sign on left side. 

Labs Positive indirect fluorescent antibody test for toxoplasmosis; 
positive Sabin-Feldman dye test. 

Imaging MR/CT. head: single or multiple rounded mass lesions with ring 
or nodular enhancement. 

Gross Pathology Large brain abscesses with concomitant focal neurologic abnor- 
malities, seizures, or altered mental status. 

Micro Pathology Parasites appear in tissue as tachyzoites or encysted bradyzoites; 
aggregates of nonencapsulated organisms constitute pseudocysts. 

Treatment Pyrimethamine; sulfadiazine. 

Discussion The definitive host of Toxoplasma gondii is the domestic cat. The 
Intermediate hosts are many and varied, including humans. 
Toxoplasmosis is also transmitted by ingestion of raw or 
undercooked meal. 



Atlas Links I'lcran M-M2-065 



HICM 2 1 MC-191 



TOXOPLASMOSIS 



e~> 



1/1 

o 
i— i 
i/i 
m 
3> 



ID/CC A 50-year-old man presents with generalized myalgia and a 
persisteni low-grade fever. 

HPI In addition, the patient recalled having severe abdominal pain 
and diarrhea several weeks ago. The patient worked in a pig 
slaughterhouse for many years. 

PE VS: fever. PE: periorbital and facial edema; tenderness over calf, 
thigh, and shoulder muscles; conjunctival and splinter hemor- 
rhages; no neurologic deficit seen. 

Labs CBC: eosinophilia. Normal ESR; elevated serum CPK, LDH, 
and AST; latex agglutination test positive for THchinelia. 

Gross Pathology Facial, neck, biceps, lower back, and diaphragm most frequently 
affected muscles. 

Micro Pathology Biopsy of sternocleidomastoid muscle reveals cysts of Trichinella 
spiralis. 

Treatment Albendazole; mebendazole; high-dose corticosteroids. 

Discussion The organism causing trichinosis. Trichinella spiralis, can be 
t ransmi tted when raw or undercooked pork is ingested. The 
larvae develop only in striated muscle cells. 

Atlas Link HJcLI 1 1 M-M2-066 






TRICHINOSIS 



ID/CC A 6-year-old male is brought lo the ER in a delirious state with 

fever and marked dyspnea thai have rapidly progressed over the 
past 2 days. 

HPI 1 lis mother, an Asian immigrant, was diagnosed and treated for 
pulmonary tuberculosis a few months ago. Me has had a low- 
grade fever, cough, malaise, and night sweats for the past 2 
months. The child has not received prophylactic isonia/.id or 
BCG vaccination. 

PE VS: fever; tachycardia: marked tachypnea; hypotension. PE: 

toxic and stuporous; pallor; central cyanosis; extensive rales and 
rhonchi bilaterally; hepatosplenomei'aly; lyinphadenopathy; 
lunduscopy reveals choroidal tubercles. 

Labs CBC: lymphocytosis; normochromic, normocytic anemia. 

Increased ESR; Mantoux skin test negative (false negative may 
occur during incubation and with severe ilisease); staining and 
culture of transbronchial and bone marrow biopsy specimens 
reveal presence of Mycobacterium tuberculosis; PCR for tuber- 
i iilosis positive; EI.ISA for I IIV negative. 

Imaging CXR: soft, uniformly distributed fine nodules throughout both 
lung Fields (miliary mottlinc;) . 

Gross Pathology Myriad 1- to 2-mm granulomas demonstrable in lungs, liver, and 
bone marrow biopsy specimens. 

Micro Pathology Granulomas with central caseous necrosis surrounded by 

epithelial cells, Langerhan's cells, lymphocytes, plasma cells, 
and fibroblasts in affected (issues. 

Treatment Multidrug antitubercular therapy with isoniazid, rifampicin, 

pyia/inamide, and ethambutol or streptomycin; steroids may be 
indicated. 

Discussion Miliary tuberculosis results from widespread hematogenous 
dissemination and often presents with a perplexing fever, 
dyspnea, anemia, and splenomegaly; the disease is more fulminant 
in children than in adults. 

Atlas Link ujcvj 1 1 PG-M2-067 



t— i 

— t 
t— t 

o 

cz 
on 

O 

on 

m 
J> 
on 



TUBERCULOSIS— MILIARY 



ID/CC A 14-year-old male immigrant complains of malaise, weight loss, 
fever, and night sweats of 6 weeks' duration; he also has a mild 
cough that began to produce bloody sputum 3 days prior to his 
admission. 

HPI The patient's mother has been diagnosed with pulmonary 
tuberculosis and is currently receiving treatment for it. 

PE VS: mild fever. PE: malnourished; low height and weight for age; 
bronchial breath sounds with crepitant rales heard over right 
supramammary area. 

Labs CBC/PBS: normocytic, normochromic anemia; WBC count 
normal with relative lymphocytosis. Increased ESR; sputum 
stained with ZN stain positive for acid-fast bacilli; positive radio- 
metric culture for Mycobacterium tuberculosis; posidve ELISA for 
TB; posidve intradermal tuberculin injection (Mantoux TEST). 

CXR: small cavity with streaky infiltrates in right upper lobe; 
hilar lymphadenopathy; calcified lung lesion (Ghon's LESION); 
Ghon's lesion and calcified lymph node (Ranke complex). 

Primary tuberculosis usually consists of lesions in lower lung 
lobes and in subpleural locations; cavitation rare; secondary TB 
or reinfection characterized by cavitary lesions usually located in 
apical regions. 

Multinucleated epithelioid Langerhan's cells surround core of 
ensealing necrosis in lung parenchyma, producing fibroblastic 
reaction at periphery with lymphocytic infiltration and 
proliferation (TUBERCLE) . 

Treatment Multiple drug dierapy with Lsoniazid (INH), rifampin, ethambutol, 
pyrazinamide, and/or streptomycin. 



Imaging 



Gross Pathology 



Micro Pathology 



Discussion 



Atlas Links 



Pulmonary tuberculosis is caused by Mycobacterium tuberculosis, 
an acid-fast, gram-positive aerobic bacillus. An increasing 
incidence in AIDS patients has been observed; drug resistance is 
becoming common. 

LJ I . M-M2-068A, M-M2-068B, M-M2-068C, PG-M2-068A, 



PG-M2-068B 



E3 



TUBERCULOSIS— PULMONARY 






ID/CC A 12-year-old white male is brought to his pediatrician because 
of an ulcer on his right wrist together with swelling of the lymph 
nodes in the right axillae with suppuration. 

HPI He had just returned from summer camp and, upon questioning, 
admits to having played with rabbits at the camp's breeding 
grounds. He has been suffering from fever, headache, and muscle 
aches for almost a week. 

PE VS: fever. PE: indurated erythematous nodule with ulcer 

formation on right wrist; right axillary adenopathy with pus 
formation; lymphangitis; mild splenomegaly; scattered rales in 
both lung bases. 

Labs CBC: normal WBC count. Increased ESR; elevated C-reactive 

protein; positive agglutination test; Frana sella lularensis on direct 
fluorescent antibody staining of material from ulcer. 

Imaging CXR: bilateral basilar interstitial infiltrates. 



Gross Pathology 



Micro Pathology 



Enlarged, indurated lymph nodes with necrosis and suppura- 
tion; skin nodule at site of inoculation with ulcer formation. 

Necrosis and suppuration of lymph nodes; pulmonary and 
disseminated lesions; granulomatous nodules with central 
caseating necrosis. 



Treatment Streptomycin and gentamicin. 



Discussion 



Tularemia is an acute zoonosis caused by Francisella tularensis, 
a nonmotile, aerobic, gram-negative bacillus; it is transmitted 
through contact with rabbits, squirrels, or other rodents or tick 
biles. It may be ulceroglandular, tonsillar, oculoglandular. 
pneumonitic, or typhoidal. 






o 

c 

o 
I— I 

m 
> 



TULAREMIA 






ID/CC A 27-year-old male is admitted to the hospital for evaluation of 
increasing fever of unknown origin along with malaise, 
headache, sore throat, cough, and constipation. 

HPI He visited Southeast Asia 3 weeks ago but did not receive any 
prior vaccinations. 

PE VS: bradycardia; fever; fever charting reveals "stepladder" 
pattern. PE: mild hepatosplenomegaly; faint erythematous 
macules seen over trunk ("rose spots"). 

Labs CBC: neutropenia with relative lymphocytosis. Widal's test 
positive in significant tilers; blood and stool cultures reveal 
Salmonella typhi. 

Gross Pathology Infection of Peyer's patches in terminal ileum leads to necrosis 
of underlying mucosa, producing longitudinal oval ulcerations. 

Micro Pathology Ulcers bordered by mononuclear cells; typhoid nodules with 
lymphocytes and macrophages may be present in liver, spleen, 
and lymph nodes. 

Treatment Ciprofloxacin is curative. 



Discussion 



Because infection is acquired from contaminated food or water, 
typhoid vaccine is recommended for all those traveling to areas 
that have had typhoid epidemics. Three vaccines are available: 
the parenteral vaccine containing the capsular polysaccharide 
and the oral vaccine containing live attenuated organisms are 
more effective than the parenteral vaccine containing whole 
killed organisms. S. typhi is transmitted only by humans, whereas 
all other Salmonella species have an animal as well as a human 
reservoir. 



Atlas Link 



IM2-030 



TYPHOID FEVER 



ID/CC A 19-year-old male goes lo his health clinic complaining of 
painful urination and discharge. 

HPI The patient had casual sex with a classmate while at a party 
2 weeks ago. He has had no previous STDs. 

PE Watery yellowish-green discharge from meatus; no penile 
ulcerations or inguinal lymphadenopathy. 

Labs Numerous neutrophils but no bacteria on Gram stain of 

discharge; positive direct immunofluorescence using mono- 
clonal antibody against Chlamydia; routine bacterial cultures, 
including Thayer-Martin, do not show growth. 

Treatment Tetracycline; doxycycline; azithromycin; treat both patient and 
sexual partner. 

Discussion The most common cause of nongonococcal urethritis is 

(Chlamydia trachomatis; less frequently it is caused by Urea-plasma 
iimilyticum. It is frequently coincident with gonococcal urethritis. 

Atlas Link mcra 1 1 M-M2-071 



URETHRITIS— NONGONOCOCCAL 



ID/CC A 25-year-old sexually active female complains of burning on 
urination. 

HPI She also complains of pain in the lower abdomen and increased 
frequency of urination. 

PE Mild suprapubic tenderness. 

Labs UA: mild proteinuria; hematuria: WBCs but no casts seen. Urine 
culture reveals > 100,000 Escherichia coli organisms present. 

Gross Pathology Infection ascends the urinary tract (urethritis, cystitis, 
pyelonephritis); mucosal hyperemia and edema. 

Micro Pathology Urothelial hyperplasia and metaplasia. 

Treatment Ciprofloxacin. 

Discussion Eighty percent of UTIs are caused by E. coli; Staphylococcus 

saprophytics is the second most common cause. Other causes, in 
order of frequency, are Proteus, Klebsiella, Enterobacter, Serratia, 
Pseudomonas, and Enterococcus; Chlamydia and Neisseria are also 
causes of urethritis. Risk factors include female gender, sexual 
activity, pregnancy, obstruction, bladder dysfunction, 
vesicoureteral reflux, and catheterization. 









URINARY TRACT INFECTION (UTI) 



ID/CC A 25-year-old sexually active woman presents with burning 

during micturition (dysuria), increased frequency and urgency 
of micturition, and low-grade fever. 

HPI She is otherwise in perfect health. 

PE VS: fever. 

Labs UA: abundant WBCs; mild proteinuria but no casts; staining of 
sediment reveals presence of gram-positive cocci. Urine culture 
isolates coagulase-negative Staphylococcus saprophyticus. 

Treatment Antibiotics (ampicillin, cotrimoxazole, or ciprofloxacin). 

Discussion Enterobacteriaceae such as Escherichia coli, Klebsiella species, and 
Proteus and Pseudomonas species are die most common organisms 
causing UTI. After E. coli. Staphylococcus saprophyticus is die most 
common cause of primary nonobstructive UTI in sexually active 
young women. 



■ • i 

5 



UTI WITH STAPHYLOCOCCUS SAPROPHYTICUS 



ID/CC A 5-year-old male presents with malaise, anorexia, fever, and a 
pruritic rash on his scalp, face, and trunk. 

HPI He also complains of a headache. Six of his classmates recently 
missed school because of similar symptoms. 

PE VS: fever (39°C). PE: skin lesions consist of macules, papules, vesi- 
cles, pustules, and scabs, all present at same time, predominantly 
over trunk, face, and scalp. 

Labs Multinucleated giant cells on scraping samples from vesicles. 
C.BC: leukopenia. 



Gross Pathology 



Micro Pathology 



Macular, papular, vesicular, and pustular rash with scab 
formation; characteristically, all lesions present at same time (vs. 
variola) ; lesions appear in crops every 3 to 5 days; myocarditis 
and pneumonitis may be present. 

Intranuclear, acidophilic inclusion bodies (Lipschutz bodies) in 
epithelial cells with clear halo around them and multinucleated 
giant cells on histologic exam of skin lesions (on Tzanck smear) . 



Treatment Acetaminophen; antihistamines and calamine lotion; hygienic 
measures, including isolation. 

Discussion A highly contagious dermotropic viral disease caused by 
varicella-zoster virus, a DNA herpesvirus, chickenpox is 
transmitted by respiratory aerosol or by direct contact. 
Complications include secondary bacterial infection of the skin 
and pneumonia; high-risk individuals may be protected passively 
with immunoglobulin and/or acyclovir. 



Atlas Links □ □ M-M2-074 



L_l i J PED-035 



VARICELLA (CHICKENPOX) 





















ID/CC An 8-year-old male is brought to a physician with complaints of 
impairment of vision in the left eye, urticarial skin rashes, and 
ill-defined muscle aches. 

HPI The child's mother has caught the child eating dirt or soil on 
many occasions (l'K \). The family also has a pel dog ai home. 

PE Rounded swelling near the optic disk seen on fundus exam of 
left eye: urticarial wheals observed on extremities and trunk; 
mild hepatosplenomegaly noted. 

Labs Leukocytosis with marked eosinophilia; enzyme immunoassay 

using extracts of excretory-secretory products of Toxocara canis 
larvae positive. 



Micro Pathology 



Treatment 



Discussion 



Biopsy of liver reveals larvae with granuloma and eosinophilic 
infiltration. 

Diethylcarbamazine; albendazole or mebendazole; steroids to 
control symptomatic inflammatory response; laser photocoagu- 
lation of visible ocular larvae. 

When the nondefinitive human host is infected with parasites 
that normally infect animals, the parasites do not mature com- 
pletely, but the larvae introduced persist and induce an inflam- 
matory reaction. The syndrome of visceral larva migrans 
develops when nematode larvae of animal parasites (mostly cat 
or dog ascarids such as Toxocara canis) migrate in human tissues; 
the syndrome ol cutaneous larva migrans (creeping eruption) 
develops when the larvae of various parasites (including the dog 
or cat hookworm Ancylostoma hmzilinise) penetrate human skin 
and form pruritic, serpiginous cutaneous lesions along the mi- 
gratory tracts of the larvae. 



<-> 

—I 
i — i 
o 
cr 
co 

o 
i— i 
en 

m 

:> 

on 



VISCERAL LARVA MIGRANS 



ID/CC A 2-year-old female is brought to the emergency room because 
of paroxysms and multiple coughs in a single expiration, fol- 
lowed by a high-pitched inspiratory whisde or whoop. 

HPI For the past 2 weeks she has had a runny nose, low-grade fever, 
muscle pains, and headache. Her immunization schedule is 
incomplete. 

PE VS: fever. PE: child apprehensive and becomes cyanotic during 
cough paroxysm; thick green mucus expelled with cough; con- 
junctival injection. 

Labs CBC: marked leukocytosis with lymphocytosis. Bardetella pertussis 
on fluorescent antibody staining of nasopharyngeal secretions; 
diagnosis confirmed by culture on Bordet-Gengou medium. 

Gross Pathology Small conjunctival and brain hemorrhages may appear during 
paroxysms; bronchiectasis may also be a complication. 



Micro Pathology 



Signs of acute inflammation in upper respiratory tract mucosa, 
with erythema, petechiae, polymorphonuclear infiltrate, and 
necrosis. 



Treatment Largely supportive; erythromycin. 

Discussion A bacterial infection of the upper respiratory tract caused by 
Bordetella pertussis, a gram-negative coccobacillus, whooping 
cough is transmitted by droplets and comprises three stages: 
prodromal (catarrhal), paroxysmal (coughing), and convales- 
cent. It is largely preventable with universally administered diph- 
theria toxoid, tetanus toxoid, and pertussis a cellular (DTP) 
vaccine. Pertussis toxin is a heat-labile exotoxin in which ADP 
ribosylates the inhibitory G protein, thus inactivating it and 
leading to constant activation of adenylate cyclase and increased 
cAMP. The remarkable lymphocytosis is due to pertussis toxin 
inhibiting chemokine receptors. As a result, lymphocytes are 
unable to leave the blood stream. 



WHOOPING COUGH 



ID/CC A 10-year-old child who lives in tropical Africa presents with 
multiple papillomatous skin lesions and pain in both legs. 

HPI The first lesion had appeared on die leg as a small indurated 
papule that ulcerated into a granulomatous papilloma. 

PE Multiple papillomatous skin lesions seen, especially in interlrigi- 

n mi is areas: lesions were painless and exuding .1 serous fluid; 
painful hyperkeratotic lesions seen on palms and soles: both 
dbia were tender to palpation. 

Labs Dark-field microscopic examination of exudate from lesions 
established the diagnosis by revealing organisms with the 
characteristic morphology and rotational motion of pathogenic 
treponemes; nonlreponemal serologic tests (i.e.. VDRL and RPR 
tests) and treponemal tests (i.e., FTA-ABS test) were positive. 

Imaging XR, legs: evidence of periostitis of the tibia. 

Treatment Long-acting intramuscular benzatiiine penicillin G is the treat- 
men I of choice. 



Discussion 



Yaws, the most common of the nonvenereal treponeniatoses, is a 
chronic infection of skin and bones caused by livpoiirma 
pertenue. Yaws occurs in tropical areas of Africa. Asia, and 
Central and South America; ii is principalh a disease of child- 
hood, and initial infection occurs between 5 and 15 years of age. 
transmission is b\ direct contact with infected skin lesions con- 
taining treponemes and is fostered by conditions of overcrowd- 
ing and poor hygiene. The disease may occur in three stages: 
primary, secondary, and tertiary. Only lesions of primary and 
secondary yaws are infectious. 



YAWS 



o 

—I 
t— I 

o 

t= 

Ln 
O 

m 
> 

m 









ID/CC 



HPI 



A 24-year-old while South American male develops sudden high 
fever, chills, generalized aches and pains, retro-orbital 
headache, nausea, and vomiting. 

He gradually improves, but the fever returns 4 days later along 
with a yellowing of his skin and eyes and an episode of fainting 
and abundant coffee-ground emesis. 



PE VS: fever (39°C); hypotension (BP 90/60). PE: jaundice; pe- 
techiae on lower legs; swollen, bleeding gums; cardiomegaly; 
hepatomegaly. 

Labs CBC: leukopenia. UA: oliguria; albuminuria; hematuria. 

Gross Pathology Normal-sized liver with yellowish hue and petechiae: pale, 
swollen kidneys. 

Micro Pathology Characteristic midzonal lobular necrosis, fatty accumulation, 
and eosinophilic inlracytoplasmic Councilman bodies on liver 
biopsy; hyperplasia of endothelial cells surrounding lymphoid 
follicles ol spleen; severe renal tubular damage with epithelial 
fatty degeneration and necrosis. 

Treatment Symptomatic; prevention with mosquito control and live viral 
vaccination. 

Discussion Yellow fever is a viral hemorrhagic fever that is caused by a 

flavivirus transmitted by Aedes mosquitoes: it is preventable by a 
vaccine, which is required prior to travel to certain countries. It 
is associated with a mortality rate of 5% to 10%, but most cases 
are self-limiting and mild. It is similar to malaria but does not 
recur. 



3 YELLOW FEVER 






ID/CC A neonate died shortly after birth. 

HPI Review of the medical record reveals history of refusal to feed, 
an extensive maculopapular skin rash on his legs and trunk, 
respiratory distress, diarrhea, and seizures shortly after birth. 

Discussion Neonatal listeriosis may occur early or late in neonatal life. 

Infants may be acutely ill at birth and may die within hours as a 
result of disseminated listeriosis, which is also called 
granulomatosis infantiseptica. This condition is characterized 
by hepatosplenomegaly, thrombocytopenia, generalized skin 
papules, whitish pharyngeal patches, and pneumonia. Commonly, 
a stained smear of meconium will reveal gram-positive bacilli, 
suggesting the diagnosis. 

Atlas Link lUCiMU PG-M2-079 



m 
O 

I 

o 
I — 
o 

CD 

-< 



LISTERIA MENINGITIS IN THE NEWBORN 









ID/CC A 2. r )-year-old woman visits her family physician because of 
marked burning pain while urinating (UYSI'RIA). increased 
frequency of urination with small amounts of urine 
(POLLAKIURIA) , and passage of a few drops of blood-stained 
debris at the end of urination (HEMATURIA). 

HPI She got married 2 weeks ago and has just returned from her 
honeymoon. 

PE VS: no fever; BP normal. PE: no edema; no costovertebral angle 
tenderness; moderate suprapubic tenderness with urgency. 

Labs UA: urine collected in two glasses; second glass more turbid 
and blood-stained; urine sediment reveals RBCs and WBCs; 
no RBC orWBC casts: Gram stain of urine sediment reveals 
gram-negative bacilli; Escherichia coH in significant colony count 
(> 100,000) on urine culture. 

Treatment Oral antibiotics (Bactrim, fluoroquinolone); adequate 
hydration. 

Discussion E. roliis the most common pathogen: Proteus. Klebsiella, 

Staphylococcus saprophylicus, and Enterococcus are other common 
bacteria causing cystitis. Hemorrhagic cystitis may result from 
adenoviral infection. 



ACUTE CYSTITIS 



ID/CC A 28-year-old man comes to the F.R witli gradually worsening 
and now severe scrotal swelling and pain radiating to the 
inguinal area. 

HPI Hie patient has no significant medical history. He reports pain 
on urination (due to concomitant urethritis) and notes that he 
is sexually active witli multiple partners. He also notes that the 
pain is greater on standing and walking and is relieved by rest 
and elevation of the legs. 

PE VS: normal. PE: scrotal edema and erythema; right epididymis 
enlarged and tender; induration present; elevation of scrotal 
contents relieves pain (PBEHN'S SIGN). 

Labs UA: pyuria. Culture negative; biopsy of epididymis inoculated 
into cell cultures grows Chlamydia trachomatis, immunofliio- 
i escencc reveals subtype D. 

Imaging US: hypoechoic. enlarged epididymis with hyper vascularity. 

Gross Pathology Nonspecific inflammation characterized by congestion and 

edema. 

Micro Pathology Earlv stage of the infection is limited to the interstitial connec- 
tive tissue with white cell infiltration. 

Treatment Antibiotics like doxycycline, minocycline for chlamydia. Course 

of ofloxacin covers all possibilities ol causative organisms. 

Discussion Differentiate epididymitis from testicular torsion and tumor 

(scrotal ultrasonography or isotopic flow study max be needed 
for differentiating). Transmitted sexually in young adults and 
most often caused by Chlamydia trachomatis subtypes D 
through K and Neisseria gonorrhoea*: In those older than 40, 
Escherichia coli and I'seudomona.s cause most infectious. II associated 
with rectal intercourse, it may be due to Enterobacteriaceae. 






TO 
O 

I — 

o 
en 
-< 



TO 

o 
o 

-< 



EPIDIDYMITIS 



ID/CC A 15-year-old male presents with painful bilateral swelling of the 
parotid glands, left-sided scrotal pain, and fever. 

HPI Nothing in the patient's history suggests that he had childhood 
mumps. He has not received a measles-mumps-rubella (M.MR) 
vaccination. 

PE VS: fever. PE: bilateral parotid gland enlargement with oblitera- 
tion of mandibular hollow; hyperemia and edema of Stensen's 
duct (parotid duct) orifice; retroauricular lymphadenopathy; 
left-sided scrotal and testicular swelling with tenderness. 

Labs CBC: leukopenia with lymphocytosis; hyperamylasemia. Positive 
complement fixation antibodies; positive serologic enzyme im- 
munoassay (EIA) for mumps antibody (repeat test after 1 week 
to demonstrate a fourfold rise). 

Imaging Is, scrotum: increased color How and edema. 

Gross Pathology Enlarged, edematous testicle. 



Micro Pathology 



Parotid glands show perivascular mononuclear, lymphocytic, 
and plasma cell infiltrate with necrosis; ductal obstruction and 
edema; testicular interstitial edema; perivascular cerebral 
lymphocytic cuffing. 



Treatment Scrotal support; analgesics, ice packs; corticosteroids. 



Discussion 



Atlas Link 



Orchitis may be caused by bacterial infections such as Escherichia 
coli and other enterobacteria; viral infections such as mumps; 

STDs such as Chlamydia species or gonorrhea; or pathogens such 
as Mycobacterium tuberculosis. Mumps orchitis may give rise to 
sterility if bilateral. 

PG-M2-082 



ORCHITIS 



ID/CC A 10-year-old child presents with complaints of acute-onset void- 
ing o I tea-colored urine and reduced urinary output. 

HPI The child was Heated 1 week ago lot streptococcal pyoderma 
that was confirmed by culture. He also complains of puffiness 
around the eyes and mild swelling of both feel. 

PE VS: hypertension (BP 140/96); lexer, tachycardia. PE: perior- 
bital swelling; mild pitting pedal edema; no ascites or kidney- 
mass palpable. 

Labs CBC: mild leukocytosis. Elevated BUN and creatinine; elevated 
ASO titer: serum < rvoglobulins present. LA: RBC casts; 
proteinuria. C3 levels reduced in blood. 

Gross Pathology Smooth, reddish-brown cortical surface with numerous 
petechial hemorrhages. 



Micro Pathology 



Biopsy shows diffuse glomerulonephritis resulting from prolifer- 
ation of endothelial, niesangial. and epithelial cells: granular. 
"starry-sky" pattern of IgG, IgM, and C3 on immunofluores- 
cence: electron microscopy shows subepithelial "humplike" 
deposits (antigen-antibody complexes). 



Treatment Penicillin if still infected with Streptococcus-, diuretics, salt and 
water restriction and antihypertensives. 

Discussion Poststreptococcal glomerulonephritis is a classic immune 

complex-mediated entity that is associated with acute nephritic 
syndrome, which develops following infection with nephrogenic 
group A 3-hemolvtie streptococci (e.g., types 1, 4, and 12, which 
are associated with pharyngitis, and types 19, 55, and 57, which 
arc associated with impetigo). 



POSTSTREPTOCOCCAL GLOMERULONEPHRITIS 









ID/CC A 25-year-old male presents with complaints of sudden-onset 

fever and chills, urgency and burning; on micturition (nv'SURlA), 
and perineal pain. 

HPI I lis symptoms developed a day after he underwent urethral 
dilatation for a stricture. 

PE VS: fever. PE: suprapubic tenderness; rectal exam reveals asym- 
metrically swollen, firm, markedly tender, hot prostate; prostatic 
massage is avoided owing to risk of inducing bacteremia; epi- 
didymitis and extreme pain. 

Labs Examination and culture of urine and prostatic secretions reveal 
infection with Escherichia coli. 



Gross Pathology 



Edematous gland enlargement with suppuration of entire gland, 
possibly abscesses and focal areas of necrosis that have coa- 
lesced. 



Micro Pathology Initially minimal leukocytic infiltration of stroma. Later, necrosis 
of the gland may lead to gland fibrosis. 

Treatment Antibiotic therapy as directed by urine and blood culture sensi- 
tivity tests. 

Discussion Escherichia coli is the most common cause of acute prostatitis; 
mam cases follow the use of instrumentation for the urethra 
and prostate (e.g., catheterization, cystoscopy, urethral dilata- 
tion, transurethral resection). Remaining infections are caused 
by Klebsiella, Proteus, Pseudomonas, and Serratia. Among the gram 
positives, eiuenx i iccus and Staphylococcus aureus are frequent 
causative organisms. 



PROSTATITIS— ACUTE 









ID/CC A 65-year-old male complains o I recurrent burning, urgency, and 
frequency of micturition together with vague lower abdominal, 
lumbar, and perineal pain. 

HPI He also complains of a mucoid urethral discharge. He was 

previously diagnosed via ultrasound with benign prostatic hyper- 
trophy but does not report any severe symptoms of prostatism; 
his medical history reveals frequent UTIs due to Escherichia coli. 

PE VS: stable; no fever. FE: rectal exam reveals enlarged, nodular 
prostate: biopsy obtained to rule out carcinoma. 

Labs Examination and culture of expressed prostatic secretions reveal 
leukocytosis and E. coli. 

Imaging FvT/voiding cystourethrogram (to rule out underlying anatomic 
cause): normal. 

Gross Pathology Enlarged prostate with nodularity and calculi. 



Micro Pathology 



Chronic inflammation and few PMNs around glands and ducts on 
biopsy; dilated ducts containing inspissated secretions (CORPORA 

amyi.u i<a). 



Treatment Antibiotics (TMP-SMX, carbenicillin. quinolones). High fluid in- 
take and abstinence from alcohol. Recurrences are common. 

Discussion Bacterial prostatitis is usually caused by the same gram-negative 
bacilli that cause UTIs in females; 80% or more of such infec- 
tions are caused by Escherichia coli. Chronic bacterial prostatitis is 
common in elderly males with prostatic hyperplasia and is a fre- 
quent cause of recurrent UTIs in males (most antibiotics poorly 
penetrate the prostate; hence the bacteria are not totally eradi- 
cated and continuously seed the urinary tract). 



-o 

O 

I — 
O 
CI 

-< 



TO 

o 

r~ 
o 
en 
-< 



PROSTATITIS— CHRONIC 



ID/CC A 28-year-old black woman who is in her 27th week of pregnancy 
complains of right flank pain, high-grade fever, malaise, 
headac he, and dysuria. 

HPI Thus far her pregnancy has been uneventful. 

PE VS: fever. PE: no peripheral edema; right costovertebral angle 
tenderness; acutely painful fist percussion on right lumbar area 

(positive Giordano's ska). 

Labs CBC: leukocytosis with neutrophilia. UA: proteinuria; hema- 
turia; abundant WBCs and WBC casts; pyocytes on sediment; al- 
kaline pH; urine culture > 1 00,000 colonies of Esclierichia coli. 

Imaging US, renal: slightly enlarged kidney- 
Gross Pathology Kidney enlarged, edematous, and hyperemic with microab- 
scesses in medulla. 



Micro Pathology 



Pyocytes in tubules; light blue neutrophils on supravital stain 

(GLITTER cells); PMN infiltration of interstitium. 



Treatment Antibiotics according to sensitivity; ampicillin; in nonpregnant 
patients, fluoroquinolone or ampicillin and an aminoglycoside 
constitute initial treatment. 



Discussion 



An acute bacterial kidney infection caused mainly by gram- 
negative bacteria such as E. coli, Klebsiella, Proteus, and 
Eiiterobacter, acute pyelonephritis usually results from upward 
dissemination of lower urinary tract bacteria. 



Atlas Links Fl F"l PG-M2-086A, PG-M2-086B 



PYELONEPHRITIS— ACUTE 



ID/CC A 10-ycar-old female presents vviih a high fever, headache, 
vomiting, and impaired consciousness. 

HPI She suffered a generalized seizure about 45 minutes ago. She 
was previously diagnosed with cyanotic congenital heart disease 
(ventricular septal defect with Eisenmenger's syndrome). 

PE VS: fever. PE: altered sensorium: papilledema: nuchal rigidity; 
clubbing; central cyanosis; cardiac auscultation suggestive of 
YSD with severe pulmonan arierial hvpertension. 

Labs Blood culture reveals mixed infection will) Bacteroides, mi- 

croaerophilic streptococci. Staphylococcus aureus, and Klebsiella: 
staining and culture of pus aspirated from brain abscess confirm 
p< ilvniii robial infection. 

Imaging CT (with contrast): multiple ring-enhancing lesions with low 

attenuation centers (ABSCESS) surrounding cerebral edema and 
ventricular compression. 

Gross Pathology Cavity filled with thick, liquefied pus surrounded by fibrous 
capsule of variable thickness; pericapsular zone of gliosis and 
edema. 

Central portion contains degenerated PMNs and cellular debris; 
capsule is composed of collagenous fibrous tissue with blood 
vessels and mixed inflammatory cells. 

Treatment High-dose, extended parenteral broad-spectrum antibiotic cov- 
erage; CT-directed drainage of pus. 



Micro Pathology 



Discussion brain abscesses arise secondary lo hematogenous spread from 
another infection (bronchiectasis, endocarditis), from contigu- 
ous spread from adjacent infection (chronic otitis media, mas- 
toiditis, sinusitis), or following direct implantation from trauma. 

Patients with i ongenital heart disease with right-to-lcft shunt are 
particularly predisposed because the normal filtering action of 
tin- pulmonary vasculature is lost. 



BRAIN ABSCESS 






ID/CC A 43-year-old male Mexican migrant worker visits his ophthal- 
mologist because of pain and loss of vision in his right eye. 

HPI Recently he has also suffered from severe headaches and 
projectile vomiting. 

PE Papilledema on left funduscopic exam; free-floating cyst in vit- 
reous body of right eye; chorioretinitis and disk hemorrhage; 
multiple nontendcr subcutaneous nodules. 

Labs CBC: eosinophilia. LP: lymphocytic and eosinophilic pleocytosis 
in CSF with elevated protein and decreased glucose. Eggs of 
Taenia solium in stool sample. 



Imaging 



Gross Pathology 



Micro Pathology 



XR, plain: small nodular calcifications. CT/MR, brain: charac- 
teristic ring-enhancing intracranial cysts or calcifications: can 
cause obstruction and hydrocephalus. 

Fluid-filled cysts containing scolex surrounded by fibrous cap- 
sule in anterior chamber of eye; intraventricular and parenchy- 
mal invasion of brain, subcutaneous tissue, and striated muscle. 

Inflammatory infiltration of cyst by PMNs; necrotic inflamma- 
tion with calcification upon death of parasite. 

Treatment Surgical removal of parasite from eye; albendazole, cortico- 
steroids/praziquantel for brain disease. 

Discussion Produced by Cystirerciis rellulosae, ihe larval form of the pork 

tapeworm Taenia solium, neurocysticercosis is due to the inges- 
tion of ova and spreads through fecal-oral transmission. 



CYSTICERCOSIS 












ID/CC A 30-year-old male presents with a high fever and chills, 
headache, nausea, vomiting, and muscle aches. 

HPI Yesterday he had an episode involving abnormal movements of 
his right hand and face (FOCAL seizure) . I le also has difficulty 
comprehending speech and has olfactory hallucinations. He has 
no history of psychiatric illness. 

PE VS: fever; tachycardia; mild tachypnea: BP normal. PE: confused 
and disoriented; papilledema: mild nuchal rigidity; Kernig's sign 
positive; paraphasic errors in speech; deep tendon reflexes nor- 
mal and bilaterally symmetric. 



Labs 



Imaging 



Gross Pathology 



Micro Pathology 



l.P: cells 400/uL with mononuclear pleocytosis; mildly elevated 
protein; normal glucose; CSF PCR reveals herpes simplex virus 
type 1 (HSV-1); serum complement-fixing antibody tiler > 
1:1000. EF.G: spiked and slow waves localized to temporal lobes. 

CT: characteristic changes of encephalitis seen over temporal 
lobes. 

Hemorrhagic, necrotizing encephalitis most severe along infe- 
rior and medial regions of temporal lobes and orbitofrontal 
gvri. 

Brain biopsy reveals Cowdry intranuclear viral inclusion bodies 
in both neurons and glial cells with perivascular inflammatory 
infiltrates. 



Treatment Intravenous acyclovir. 

Discussion Herpes simplex virus is the most common cause of acute 

sporadic encephalitis in the United States. In the newborn, 
HSV-2 is usually the cause; after the neonatal period, most cases 
result from HSV-1. Neonatal infection (usually HSV-2) occurs 
after exposure to maternal genital infection at the time of 
delivery. The precise pathogenesis of HSV-1 encephalitis in the 
older child or die adult is not clear, but viral spread into the 
temporal loin- l>\ both nlfac Ion and trigeminal routes has been 
postulated. 

Atlas Link UlcHI 1 1 M-M2-089 



HERPES SIMPLEX ENCEPHALITIS 






ID/CC An 1 1-year-old girl is brought to the ER with high fever, chills, 
severe headache, vomiting, and obtundation. 

HPI Her parents report that she suffered a generalized seizure about 
an hour ago. A few days ago. the family had returned from a 
.summer vacation in south India, where the child often played in 
irrigated rice farms. She did not receive any immunizations 
prior to her travel. 

PE VS: fever. PE: patient is stuporous; neck stiffness and Kernig's 
sign positive (due to meningeal irritation): mild papilledema; 
tremors noted in hands. 

Labs LP: CSF reveals pleocytosis with predominant lymphocytosis, 

mildly elevated proteins, and normal sugar (suggestive of aseptic 
meningitis); IgM enzyme immunoassay performed on acute and 
convalescent sera and CSF reveals significant titer of antibodies 
to Japanese encephalitis virus. 

Imaging ( T. head: areas of low density in the thalamus and basal ganglia. 

Treatment Supportive; experimental intrathecal a-interferon therapy. 

Discussion Japanese encephalitis virus is a flavivirus that causes disease in 
humans, horses and pigs. It is widely distributed in Asia from 
Japan and Eastern Siberia to Indonesia and westward to India; 
epidemics occur in summer months coincident with the abun- 
dance of the mosquito vector Culex tritaenioHiychnus. The vec- 
tor breeds in irrigated rice fields and bites preferentially at 
sunset and sunrise; pigs are the amplifying hosts, whereas man is 
the incidental "dead-end" host. A vaccine is available for routine 
use for childhood immunization in Japan and in developed 
counuies to protect travelers. 



JAPANESE ENCEPHALITIS 






ID/CC A 30-year-old male laboratory researcher presents with a high 
fever, neck rigidity, retro-orbital pain, and severe myalgias of a 
few days' duration. 

HPI The patient also complains of a sore throat and photophobia. 

His work in the lab involves close contact with experimental ani- 
mals such as hamsters, white mice, and nude mice. He was ade- 
quately vaccinated. 

PE VS: fever. PE: neck stiffness and Kernig's sign positive (due to 
meningeal irritation); pharyngeal inflammation but no exudate 
noted. 

Labs CBC: mild leukopenia. LP: CSF suggestive of aseptic meningitis; 
LCM virus isolated from CSF. IgG and IgM antibodies detected 
in serum by immunofluorescent assay. 

Treatment Supportive; ribavirin may play a role. 

Discussion Lymphocytic choriomeningitis virus is an arenavirus. Sporadic 
cases occur after infection with feral mice, but the most 
common sources of human infection are pet/lab rodents. The 
virus is considered a major lab hazard, and care must be taken 
to avoid accidental infection. There is no commercially available 
vaccine. 



LYMPHOCYTIC CHORIOMENINGITIS (LCM) 









ID/CC A 50-year-old white male develops sudden fever with chills, pain 
in the back and extremities, and neck stiffness; he vomited six 
limes and had a convulsion prior to admission. 

HPI The patient is a heavy smoker and is diabetic. Two weeks ago, 

he had a URL He is also very sensitive to light (photophobia). 

PE Markedly reduced mental status (obtunded); petechial rash 
over trunk and abdomen; nuchal and spinal rigidity; positive 
Kernig's and Brudzinski's signs; no local neurologic deficits. 

Labs LP: elevated pressure; cloudy CSF; elevated protein; markedly 

decreased glucose; high cell count with mostly WBCs. CSF Gram 
stain reveals gram-positive diplococci. Spinal fluid culture grows 
Streptococcus pneumoniae. 

Imaging CT/MR, brain: meningeal thickening and enhancement. 



Gross Pathology 



Pia-arachnoid congestion results from inflammatory infiltrate; 
thin layer of pus forms and promotes adhesions while obstruct- 
ing normal CSF flow (can cause hydrocephalus); brain covered 
with purulent exudate, most heavily on base. 



Treatment Early empiric high-dose IV antibiotics; cefotaxime: vancomycin; 
high-dose steroids. 

Discussion Bacterial meningitis is a pyogenic infection of the CNS that re- 
quires prompt treatment. Streptococcus pneumoniae is the most 
common cause of adult meningitis. 

Atlas Links QEH j ] PG-M2-092A, PG-M2-092B, M-M2-092 

Cli. □' NEU-027 



MENINGITIS— BACTERIAL (ADULT) 



ID/CC A 4-year-old female presents with a 1-week history of fever, se- 
vere headache, irritability, and malaise: 2 days ago she devel- 
oped neck stiffness, and her parents report projectile vomiting 
over the past 24 hours. 

HPI The child is also very sensitive to light (photophobia). She is 
fully immunized and has no history of ear, nose, and throat 
infection, skin rashes, dog bites, or foreign travel. 

PE VS: fever. PE: irritability; resistance to being louched or moved; 
minima] papilledema of fundus; no focal neurologic signs; no 
cranial nerve deficits; positive Kernig's and Brudzinski's signs. 

Labs CBC: neutrophilic leukocytosis. LP: increased pressure; cloudy 
CSF; neutrophilic pleocytosis; decreased glucose; increased 
protein; gram-negative coccobacilli. Negative ZN and India ink 
staining; normal scrum electrolytes; on chocolate agar, blood 
culture grew Haemophilus influenzae; negative Mantoux. 

Imaging CT/MR, brain: meningeal thickening and enhancement. 

Gross Pathology Abundant accumulation of purulent exudate between pia mater 
and arachnoid; meningeal thickening; cloudy to frankly 
purulent CSF. 

Micro Pathology Intense neutrophilic infiltrate. 

Treatment IV antibiotics (ampicillin, cefotaxime); consider steroids. 

Discussion A pyogenic infection of the nervous system primarily affecting 
the meninges, bacterial meningitis is most commonly caused by 
pneumococcus (Streptococcus pneumoniae, associated with sickle 
cell anemia), meningococcus (Neisseria meningitidis, associated 
with a petechial skin rash), and H. influenzae (most commonly in 
children). It is less commonly caused by enterobacteria, 
Streptococcus species, Staphylococcus species (due to dental infec- 
tion), and anaerobic organisms (clue to trauma). 

Atlas Link mcfc'j 1 1 M-M2-093 



TO 

o 
o 

CI 

-< 






?| MENINGITIS— BACTERIAL (PEDIATRIC) 



ID/CC A 33-year-old HIV-positive white male is brought into the emer- 
gency room by his mother because of a persistent headache. 

HPI The patient's mother states that her son has been suffering for a 
long tune from headaches and stiff neck as well as from fever 
and chills. 

PE VS: fever (39°C). PE: severe nuchal rigidity; lack of responsive- 
ness 10 any command; positive Kemig's and Rrud/iiiski's signs; 
diminished patellar and Achilles reflexes; clear lung sounds. 

Labs LP: increased CSF pressure; variable pleocytosis (75 lymphocytes/ 
mm 1 ); elevated protein; decreased glucose. Heavily 
encapsulated, nondimorphic spherical fungal cells 
(CRYPTOC.occ.rs \t.oi-or\ia\s) revealed on India ink staining; 
polysaccharide capsular antigen detected on latex agglutination 
test; diagnosis confirmed by culture on Sabouraud's medium. 

Imaging CT/MR. brain: multiple ring-enhancing lesions. 



Gross Pathology 



Micro Pathology 



Granuloma and abscess formation, mainly at base of brain; CNS 
primarily affected; lungs affected less commonly. 

Abundant fungi in CSF and leptomeninges, with slight mononu- 
clear inflammatory reaction; typical nodular granulomatous 
meningitis with exudate. 



Treatment Amphotericin B and 5-flucytosine; fluconazole. 

Discussion Once called torulosis, cryptococcosis is the most common cause 
of mycotic meningitis; it is acquired through the inhalation of 
dried pigeon droppings and is usually seen in immunocompro- 
mised patients. 

Atlas Link L'JCL-a 1 1 M-M2-094 



MENINGITIS— CRYPTOCOCCAL 



ID/CC A 6-year-old male being treated for primary pulmonary 

tuberculosis presents with diplopia, increasing drowsiness, 
irritability, and unexplained, recurrent vomiting. 

HPI The child has had a low-grade fever, loss of appetite, and a per- 
sistent headache over the past few weeks. 

PE VS: fever. PE: stuporous; signs of meningeal irritation noted 

(neck rigidity, Kernig's sign); CN III and IV palsy on right side; 
lunduscopy reveals papilledema. 

Labs LP (guarded): CSF under increased pressure and turbid; on 

standing, a "cobweb" coagulum formed at center of tube; ( !SF 
studies reveal lymphocytic pleocytosis. greatly elevated protein, 
and low sugar; ZN staining of CSF coagulum reveals presence of 
acid-fast bacilli; radiometric culture yields Mycobacterium 
tuberculosis. 



Imaging 



Gross Pathology 



Micro Pathology 



CT: suggests basal exudates, inflammatory granulomas, and a 
communicating hydrocephalus; striking meningeal enhance- 
ment noted in post-contrast studies. 

Meningeal surface covered with yellowish-gray exudates and tu- 
bercles that are most numerous at base of brain and along the 
course of the middle cerebral artery; subarachnoid space and 
arachnoid villi obliterated (leading to poor absorption of CSF 
and hence a communicating hydrocephalus). 

Subarachnoid space contains gelatinous exudate of chronic in- 
flammatory cells, obliterating cisterns, and encasing cranial 
nerves; well-formed granulomas occasionally seen, most often at 
base of brain; arteries running through subarachnoid space 
show "obliterative endarteritis." 



Treatment Anlituberculous therapy with rifampin, isoniazid, ethambutol 

and pyrazinamide; steroids; ventriculoperiloneal shunt to relieve 
hydrocephalus. 

Discussion Tuberculous infection reaches the meninges through the 

hematogenous route, resulting in a clinically subacute form of 
meningitis; it is often complicated by cranial nerve palsies, a 
communicating hydrocephalus, decerebrate posturing, 
convulsions, coma, and death. 






O 

r - 
o 

CI 

-< 



MENINGITIS— TUBERCULAR 






ID/CC A 3-year-old male, the child of recent African immigrants, is 

brought to the local health center because of asymmetrical legs. 

HPI His parents give a history of incomplete immunization. They add 
that 5 months ago the boy had fever and diarrhea that subsided 
spontaneously; a few weeks later they noted that he could not 
use his right leg. 

PE Right leg thin, short, wasted, weak, and flaccid; absent deep 
tendon reflexes in right leg; no sensory deficit; upper limbs 
normal; mental status and cranial nerves normal. 

Labs EMCi: chronic partial denervation with abnormal spontaneous 
activity in resting muscle and reduction in number of motor 
units under voluntary control; normal sensory conduction 
studies. 

Treatment Rehabilitation, supportive. 

Discussion A symptomatic disease caused by poliovirus that is more com- 
mon in infants and children, poliomyelitis can result in muscu- 
lar atrophy and skeletal deformity. It attacks motor neurons in 
the anterior horns and may affect cranial nerves (bulbar polio); 
it is preventable by vaccine. 



POLIOMYELITIS 



ID/CC A 26-year-old nurse presented with headaches and recent-onset 
seizures; she also complained of increasing right-sided 
numbness and blurring of vision. 

HPI A clinical diagnosis of HSV encephalitis had previously been 

made, for which the patient was treated with two courses of acy- 
clovir without any amelioration of symptoms; the disease contin- 
ued to progress both radiologically and clinically. On serology 
she tested HIV positive. 

PE Neurologic exam reveals cognitive mental impairment; visual 
Field defects and sensory dysphasia seen; an ill-defined sensory 
loss on right side of body. 

Labs HIV positive by ELISA and Western blot. 



Imaging 



Micro Pathology 



Treatment 



Discussion 



MR (T2-weighted): patchy high-intensity lesions in the deep 
white matter of left cerebral hemisphere involving temporal, 
parietal, and occipital lobes. 

Stereotactic biopsy sections show abnormal brain with rarefac- 
tion, numerous reactive astrocytes, foamy histiocytes, and in- 
flammatory infiltrate around some vessels; JC virus in situ 
hybridization shows many positive nuclei; no herpesvirus inclu- 
sions seen: electron microscopy demonstrates cells with typical 
papovavirus structures in nucleus. 

Disease was relentlessly progressive and resulted in death within 
6 months. 

Progressive multifocal leukoencephalopathy is a progressive 
demyelinating disease related to JC papovavirus infection; the 
largest number of cases occur in persons who are immunocom- 
promised for any of a variety of reasons, including organ trans- 
plantation, hematologic and other malignant diseases, chronic 
immunosuppressive therapy, and AIDS. 



73 

O 

O 
en 

-< 



PROGRESSIVE MULTIFOCAL LEUKOENCEPHALOPATHY 



ID/CC A 30-year-old male is seen with complaints of a rash along with 
pain in his left ear and inability to move the muscles of his face 
with accompanying asymmetry. 

HPI He suffered an attack of chickenpox during childhood but has 

lid history either of a similar rash over his face or of any visual 
symptoms (to rule out herpes zoster ophthalmicus). 

PE Vesicular rash over left pinna (otitis kxtkrna); left-sided lower 
motor neuron-type facial nerve palsy (patient is unable to frown 
and unable to blink left eye; eyeballs roll up during attempt to 
close eye; patient is unable to whistle; taste sensation over 
anterior two-thirds of tongue lost on left side). 

Labs Although the diagnosis is predominantly clinical, a Tzanck test 
examining lesion scrapings (showing evidence of multinucleate 
acanlholytic cells), direct culture, and immunohistochemical 
identification of infected cells allow identification of the virus. 

Gross Pathology Neuritis and vesicular skin lesions confined to distribution of 
geniculate ganglion of facial nerve. 



Micro Pathology 



Vesicular skin lesions widi herpes viral inclusions, i.e.. intranu- 
clear, acidophil inclusions with a halo around them (Cowdry 
i mi A inclusions); syncytial cells also seen. 



Treatment Systemic steroids are mainstay of management. 

Discussion Herpes zoster of the geniculate ganglion, or Ramsay Hunt syn- 
drome, presents as a vesicular rash on the pinna followed by 
ipsilateral I.MN facial nerve palsy. 






RAMSAY HUNT SYNDROME 



ID/CC A 30-year-old male presents with a high fever, neck stiffness, and 
drowsiness. 

HPI He also complains of nausea and vomiting. He recently traveled 
along the Mississippi-Ohio River basin. 

PE V'S: fever. PK: neck stiffness and Kernig's sign positive (due to 
meningeal irritation); right oculomotor nerve palsy noted; mild 
papilledema. 

Labs IgM enzyme immunoassay done on paired sera, and CSF con- 
firms the diagnosis of St. Louis virus infection. LP: CSF exam 
reveals pleocytosis with predominant lymphocytosis suggestive of 
aseptic meningitis. 



Micro Pathology 



Inflammation and neuronal degeneration, principally in the 
thalamus, midbrain, and brainstem. 



Treatment Supportive treatment. 

Discussion St. Louis encephalitis virus is the most common cause of 

epidemic encephalitis in the United Stales: cases occur annuallv 
as isolated events or in summer and autumn encephalitis 
epidemics. Most infections are asymptomatic. The disease occurs 

throughout the United States, but outbreaks have also occurred 
in the Caribbean as well as in Central and South America. 



cz 

TO 
O 

I — 
O 
CD 

-< 



ST. LOUIS ENCEPHALITIS 



ID/CC The case of a 12-year-old boy who died of a progressive degener- 
ative neurologic disease was discussed at an autopsy meeting. 

HPI The child had been developing normally up to 10 years of age, 
when his teachers noted a progressive deterioration in intellect 
and personality; this was followed by the development of 
seizures akin to myoclonus, signs of pyramidal and extrapyrami- 
dal disease, and finally a state of decerebrate rigidity. The child 
died 7 months after the onset of symptoms. His history revealed 
that he had had a severe attack of measles at the age of 2. 

Labs LP: routine CSF profile normal. Gamma globulin level elevated: 
markedly elevated levels of measles antibody present in both 
serum and CSF; despite the elevated antibody liters, antibody to 
the M protein was not present. EEC: pattern of burst 
suppression and biphasic sharp and slow waves. 

Imaging MR: nonspecific parenchymal abnormalities. 



Micro Pathology 



Histopathologically. the encephalitis involved both white and 
gray matter and was marked by lymphocytic infiltration, nerve 
cell degeneration, and demyelination; measles antigen demon- 
strated by immunofluorescence analysis, and particles resem- 
bling paramyxovirus were detected by electron microscopy. 



Treatment No specific therapy available. 

Discussion Subacute sclerosing panencephalitis (SSPE) is caused by a 

defective (major defect is the lack or altered expression of the 
M-matrix protein) form of measles virus (family Paramyxoviridae); 
SSPE is a late complication of a measles infection that is not 
eliminated from the host. Immunization against measles is the 
only effective preventive tool. 



SUBACUTE SCLEROSING PANENCEPHALITIS 






ID/CC A 25-year-old recently married woman is concerned about a 
scanty, offensively malodorous vaginal discharge. 

HPI She states that the discharge is thin, grayish-white, and foul- 
smelling. She does not complain of vulvar pruritus or soreness. 

PE Pelvic exam confirms presence of a homogenous, grayish-white, 
watery discharge adherent to the vaginal walls that yields a 
"fishy" odor when mixed with KOH; no injection and excoria- 
tion of the vulva, vagina, or cervix. 

Labs Vaginal pH > 5; saline smear reveals presence of characteristic 
"clue cells" (squamous epithelial cells with smudged borders 
due to adherent bacteria). 

Treatment Single dose of metronidazole (2 g) effective in treating the 
infection. Oral clindamycin is an alternative drug. 

Discussion Although bacterial vaginitis was originally thought to be caused 
by Gardnerella vaginalis, this organism is now recognized to be 
part of the normal vaginal flora. Bacterial vaginosis is now 
known to result from a synergistic interaction of bacteria in 
which the normal Lactobacillus species in the vagina is ultimately 
replaced by high concentrations of anaerobic bacteria, including 
Bacteroides, Peptostreptococcus, Peptococcus, and Mobiluncus 
species along with a markedly greater number of G. vaginalis 
organisms than is encountered in normal vaginal secretions. 
Bacterial vaginosis is known to increase the risk of pelvic inflam- 
matory disease, chorioamnionitis, and premature birth. 

Atlas Link VlcVi 1 1 M-M2-101 



BACTERIAL VAGINOSIS 









S 



o 
o 

CI 

-< 



ID/CC A 25-year-old puerpera who was lactating her week-old infant 
presents with pain and swelling in her left breast. 

HPI The symptoms commenced acutely, and she does not recall any 
previous breast lumps or swellings. 

PE Skin overlying left breast is red, edematous, tender, and hot; 
area of tense induration felt underlying inflamed skin. 

Labs Culture of pus drained from breast abscess and nasopharyngeal 
swab taken from the infant grew Staphylococcus aureus. 

Imaging USG: nearly anechoic area with posterior enhancement. 

Treatment Penicillinase-resistant antibiotic; incision (in a radial direction 
over the affected segment) and dependent drainage of intra- 
mammary abscess; breast feeding was temporarily discontinued. 

Discussion Bacterial mastitis most commonly occurs in lactating women 
due to infection of a hematoma or secondary infection of 
plasma cell mastitis; the infecting organism is mostly 
penicillin-resistant Staphylococcus aureus. 



BREAST ABSCESS 



ID/CC A 38-year-old while female visits her gynecologist for a routine 
Pap smear. 

HPI She admits to early sexual activity, many sexual partners, and 
unprotected sex. 

PE Pallor; cervical tenderness; a few small, raised, flat lesions on 
cervix: genital warts also seen on vulva (condylomata 
a< i WIN ATA). 

Labs Presence of HPV in cervical cells revealed on DNA hybridization 
and immunofluorescent antibody assays for viral antigen. 



Micro Pathology 



Rounded basophilic cells on Pap smear with large nuclei occupy- 
ing most of surface; nuclei show mitoses and coarse clumping of 
chromatin with perinuclear halo (severe KOILOCYTIC DYSPLASIA). 



Treatment Cryotherapy, conization, or local excision with follow-up. 

Discussion Infection with HPV types 16, 18, and 31 is strongly associated 
with cervical cancer preceded by dysplasia. Spread of die infec- 
tion to partners may be prevented by barrier contraception. 



Atlas Links Ul< IT ; M-M2-103 



L»JCk'i21 0B-011A, 0B-011B 



HUMAN PAPILLOMAVIRUS (HPV) 



ID/CC A 28-year-old sexually active woman presents with crampy lower 
abdominal pain, yellowish vaginal discharge, and general 
malaise. 

HPI She also complains of continuous low-grade fever and reveals 
thai the pain is exacerbated during and immediately after 
menstruation (congestive DYSMENORRHEA). She uses a copper 
intrauterine device for contraception. 

PE VS: low-grade fever. PE: lower abdominal tenderness; bimanual 
pelvic exam demonsuates purulent vaginal discharge, bilateral 
adnexal tenderness, and pain on movement of cervix 

(MUCOPURUl-ENT CERVICITIS). 

Labs CBC: leukocytosis with left shift. Increased ESR; endocervical 
swab sent for microscopic exam; staining and culture revealed 
combined infection with Neisseria gonorrhoeae (cultured on 
Thayer-Martin medium) arid Chlamydia trachomatis (identi- 
fied on cell culture, immunofluorescence, and antigen capture 
assay); laparoscopy ("gold standard" for diagnosis) confirmed 
diagnosis. 

Imaging USG: free pelvic fluid, dilated tubular structure in adnexa. 

Gross Pathology Erythema and swelling of fallopian tubes on laparoscopy; sero- 
purulent exudate noted on surface of tubes from Gmbriated end. 



Micro Pathology 



Treatment 



Discussion 



Endocervical swab reveals increased neutrophils and gram- 
negative diplococci seen both intra- and extracellularly; cervical 
biopsy reveals inclusions containing Chlamydia within columnar 
cells. 

Antibiotic therapy with cefoxitin (for N. gonorrhoeae) and doxycy- 
cline (for chlamydial infection); male partners must be treated 
for STDs. 

Pelvic inflammatory disease usually occurs as a primary infec- 
tion that ascends from the lower genital tract due to STDs 
caused by Neisseria gonorrhoeae and Chlamydia trachomatis. 
Sequelae of PID include peritonitis; intestinal obstruction due 
to adhesions; dissemination leading to arthritis, meningitis, and 
endocarditis; chronic pelvic pain; infertility; ectopic pregnancy; 
and recurrent PID. 



PELVIC INFLAMMATORY DISEASE 









ID/CC A 20-year-old Asian woman presents with complaints of 

infertility and heavy bleeding during menses (MENORRHAGIA). 

HPI She was treated for pulmonary tuberculosis a few years ago. She 
has been unable to conceive despite unprotected intercourse for 
the past 2 years. Her husband's semen analysis is normal. 

PE On pelvic exam, small, fixed adnexal masses are palpable that 
are matted and fixed to uterus ("FROZEN PELVIS"). 

Labs Culture of endometrial cureitings yields Mycobacterium 
tuberculosis; histologic examination of curettings reveals 
presence of characteristic tubercles; Mantoux skin test strongly 
positive. 



Imaging 



Gross Pathology 



CXR: left apical fibrosis (evidence of old healed pulmonary 
tuberculosis). (Hysterosalpingography [HSG] is contraindicated 
in a proven case of tuberculosis. When done in asymptomatic- 
cases, HSG yields certain typical findings, including a rigid, 
nonperistaltic, pipelike tube; beading and variation in tilling 
density; calcification of the tube; cornual block; jagged 
fluffiness of the tubal outline; and vascular or lymphatic 
extravasation of the dye.) 

Tubes are enlarged, thickened, and tortuous; examination of 
uterus reveals evidence of synechiae and adhesions (leading to 
Asherman's syndrome) . 



Micro Pathology Microscopic exam of tubes, ovaries, and endometrium reveals 
evidence of granulomas with giant cells and caseation. 

Treatment Four-drug therapy with isoniazid, pyrazinamide, ethambutol. 
and i ilampicin; pyridoxine to prevent isoniazid-induced 
deficiency. 

Discussion Genital tuberculosis is almost always secondary to a focus 

elsewhere in the body, with the bloodstream by far the most 
common method of spread. The fallopian tubes are the most 
frequently involved part of the genital tract, followed by the 
uterus. Ninety percent of patients are cured with chemotherapy, 
although only 10% regain fertility. 



PELVIC TUBERCULOSIS 



CD 

-< 

-z. 

m 
O 
O 

I — 

o 

en 

-< 



ID/CC A 30-year-old woman presents to the F.R with an abrupt-onset 

high fever, vomiting, profuse diarrhea, severe muscle aches, and 
disorientation. 

HPI One day ago she developed an extensive skin rash all over her 
body. Her husband says she used a vaginal sponge for 
contraception. 

PE VS: fever; tachycardia; hypotension. PE: extremely toxic-looking; 
drowsy but responding to verbal commands; extensive 
scarlatiniform rash seen involving entire body; pharyngeal, 
conjunctival, and vaginal mucosa congested (frank hyperemia); 
no neck rigidity or Kernig's sign demonstrable; funduscopic 
exam normal; no localizing neurologic deficits. 

Labs CBC: leukocytosis; thrombocytopenia. UA: mild pyuria (in 

absence of L'TI). BUN and creatinine elevated; blood cultures 
sterile; culture of cervical secretions grows Staphylococcus 
aureus. LP: CSF normal. Serology for Rocky Mountain spotted 

lever, leptospirosis, and measles negative. 

Treatment Vigorous IV fluids and parenteral penicillinase-resistant 

penicillin or first-generation cephalosporins: patient in this case 
recovered, and typical skin desquamation was seen over palms 
and soles during convalescence. 

Discussion Toxic shock syndrome results from infection with Staphylococcus 
aureus. Its effects are mediated through the exotoxin TSST-1, 
which functions as a superantigen, stimulating the production 
of interleukm-1 and tumor necrosis factor. Staphylococcal TSS 
has been associated with the use of vaginal contraceptive 
sponges and with many types of localized staphylococcal soft 
tissue infections. Most cases of TSS occur in menstruating women. 

Atlas Links QJTEIll 0B-027A, 0B-027B 






TOXIC SHOCK SYNDROME (TSS) 



ID/CC A 28-year-old primigravida al 36 weeks' gestation presents with a 
high fever. 

HPI She was being monitored following a premature rupture of the 
membranes. 

PE VS: fever; fetal tachycardia. PE: uterine tenderness. 

Labs Elevated maternal total lymphocyte count; vaginal swab culture 
revealed colonization with group B streptococcus. 

Treatment Presence of group B streptococcus in vagina after premature 

rupture of membranes was an indicadon for immediate delivery 
and treatment of the infant; mother was also treated with 
intravenous antibiotics. 

Discussion A significant proportion of the population is colonized in the 
vagina and rectum with group B streptococcus, which is 
correlated with preterm labor, premature rupture of 
membranes (PROM), chorioamnionitis, and neonatal sepsis; 

neonates widi group B streptococcus sepsis have a 25% mortality 
rate. Among preterm neonates, this figure doubles to over 50%; 
therefore antibiotic prophylaxis is recommended in the setting 
of preterm delivery and PROM even without die diagnosis of 
frank chorioamnionitis. When chorioamnionitis is suspected, 
intravenous antibiotics are started and delivery is hastened. 



CHORIOAMNIONITIS 



o 

CO 

—t 

3. 

pa 
i— i 

Q 



ID/CC 



HPI 



PE 



Labs 



Treatment 



Discussion 



An 8-month-old male infant is brought to a pediatrician because 
of severe, intractable chronic diarrhea and failure to thrive. 

The mother died of AIDS shortly after the baby was delivered. 
The baby was asymptomatic at birth. 

VS: fever; tachycardia. PE: emaciated, grossly malnourished; oral 
thrush; generalized lymphadenopathy; hepatosplenomegaly. 

Decreased CD4+ cell count; increased serum immunoglobulin 
level with impaired production of specific antibodies; ELISA and 
Western blot for HTV-1 positive (could be due to placental 
transfer of antibodies to HIV, but strongly supports diagnosis in 
presence of symptoms); PCR for HTV* RNA positive (confirming 
HIV infection). 

Nutritional support, Pneumocystis carinii prophylaxis, azidothymi- 
dine (zidovudine, or AZT) therapy (suppresses replication by 
inhibiting viral reverse transcriptase), and anti-infective agents 
for specific infections; IV serum immunoglobulin to reduce 
frequency of bacterial infections; oral polio vaccine and BCG 
contraindicated . 

Vertical transmission of HIV-l may occur in utero through 
transplacental passage of the virus, during delivery, or postnatally 
through breast feeding; however, it is believed that most 
infections are acquired at birth through contact with contami- 
nated blood or secretions. Women who carry the virus should 
thus be discouraged from becoming pregnant or from breast 
feeding. The rate of transmission of HIV-l from mother to 
infant has varied from 13% to 45%, with an average of 25%; 
however, when AZT is administered to HIV-l -infected pregnant 
women and to infants during the first 6 weeks of life, the risk of 
maternal-infant transmission is significantly reduced. 



HIV TRANSMISSION IN PREGNANCY 






ID/CC A 4-year-old while male presents with fever, chills, malaise, pain. 
and immobility of the right knee of 1 week's duration. 

HPI Two weeks ago the child fell while playing, but no abnormality 
was found by the school nurse. 

PE ( )verlying skin warm and red; swelling of distal third of thi«h 
and knee; tenderness on palpation. 

Labs CBC: leukocytosis. Elevated ESR. Gram stain and culture con- 
firm diagnosis and isolate pathogen. 

Imaging XR, plain: early findings include soft tissue edema and thin line 
running parallel to diaphysis (periosteal thickening); later find- 
ings include bone erosion, subperiosteal abscess with periostitis, 
and sequestrum formation (due to detached necrotic cortical 
bone): involucrum formation (laminated periosteal reaction). 
MR: marrow edema; abscess. Indium-labeled WBC, scan: hot 
spot. 

Gross Pathology New osteoblastic periosteal bone formation (INVOLUCRUM); 
trapping of detached necrotic bone by involucrum 

(SEQUESTRUM); isolated localized abscess (Broihe's abscess); 
sinus tract formation, draining pus to skin. 



Micro Pathology 



Purulent exudate formation, usually metaphyseal, with ischemic 
necrosis of bone due to increased pressure of pus in rigid bone 
walls; vascular thrombosis. 



Treatment IV* antibiotics according to sensitivity; surgical debridement. 

Discussion Osteomyelitis is an acute pyogenic bone infection which, if left 
untreated, produces functional incapacity and deformities. The 
most common pathogen is Staphylococcus aureus; less fre- 
quently .SV rcptoaicats and enterobacteria are involved. In sickle 
cell anemia Escherichia roli and Salmonella species are seen; diabet- 
ics are at risk for Pseudomonas infection. Immunocompromised 
patients may show Sporolhrix schenckii osteomvelitis: human bites, 
anaerobes; puncture wounds. Pseudomtmas aeruginosa; and cat- 
bile wounds. I'aslntri'lla inullocida. 



O 
*3 



o 
-a 



o 
1/1 



OSTEOMYELITIS 



ID/CC A 21-year-old female college student complains of low-grade 
fever along with pain and swelling in the left knee of 5 days' 
duration. 

HPI She had been to her family physician 2 weeks ago because of 
dysuria and a purulent vaginal discharge (due to gonococcal 
infection) and was given an "antibiotic shot". She was 
asymptomatic until 4 days ago. She then developed fever, chills. 
and pain in both wrists and in her left ankle, which disappeared 
when the pain appeared in her left knee (migratory poly- 
arthralgia) . 

PE Swollen, tender, warm left knee with limited range of motion; 

white vaginal discharge. 

Labs Intracellular, bean-shaped gram-negadve diplococci 

((.( i\( «.:()( ( ij and markedly elevated WBC count on urethra] 
smear and synovial fluid aspirate culture of synovial aspirate 
grows gonococci. 

Imaging XR, knee: soft tissue swelling. 

Treatment IV ceftriaxone. 

Discussion .Almost always accompanied by synovitis and effusion, gonococ- 
cal arthritis can rapidly destroy ardcular cartilage and is often as- 
sociated with skin rash and C5, C6, C7, and C8 complement 
deficiencies. Single joints are usually affected, most often the 
wrists, fingers, knees, and ankles. 



SEPTIC ARTHRITIS— GONOCOCCAL